You are on page 1of 90

Refraction at a Boundary

Boundary Behavior
A wave doesn't just stop when it reaches the end of the medium. Rather, a wave will undergo
certain behaviors when it encounters the end of the medium. Specifically, there will be some
reflection off the boundary and some transmission into the new medium. The transmitted
wave undergoes refraction (or bending) if it approaches the boundary at an angle. If the
boundary is merely an obstacle implanted within the medium, and if the dimensions of the
obstacle are smaller than the wavelength of the wave, then there will be very noticeable
diffraction of the wave around the object. The reflection, refraction, and diffraction of waves
were first introduced in Unit 10 of The Physics Classroom tutorial. In Unit 11 of The Physics
Classroom Tutorial, the reflection, refraction, and diffraction of sound waves was discussed.
Since light is a wave, it too will undergo these same behaviors when it reaches a boundary
between two medium. The boundary behavior of light waves has already been introduced
in Unit 12 of The Physics Classroom Tutorial. In this unit, we will focus on the refraction of
light in great detail. We will explore the conceptual and mathematical principles governing the
bending of waves as they cross the boundary between two media. To understand light
refraction, we will need to back up a few steps and investigate the behavior of waves when
they reach the end of a medium.

Boundary Behavior for Waves on a Rope

Suppose that there is a thin rope attached to a thick rope, with each rope held at opposite
ends by people. And suppose that a pulse is introduced by the person holding the end of the
thin rope. If this is the case, there will be an incident pulse traveling in the less dense medium
(thin rope) towards the boundary with a more dense medium (thick rope).

Upon reaching the boundary, two behaviors will occur.

• A portion of the energy carried by the incident pulse is reflected and returns
towards the left end of the thin rope. The disturbance which returns to the left
after bouncing off the boundary is known as the reflected pulse.
• A portion of the energy carried by the incident pulse is transmitted into the thick
rope. The disturbance which continues moving to the right is known as
thetransmitted pulse.

These two behaviors - reflection and transmission - were first introduced in Unit 10 of The
Physics Classroom. In that unit, it was mentioned that the passage of the energy from the
incident medium into the transmitted medium was accompanied by a change in speed and
wavelength. In the case of a pulse crossing the boundary from a less dense medium into a
more dense medium, the speed and the wavelength are both decreased. On the other hand,
if a pulse crosses the boundary from a more dense medium into a less dense medium, the
speed and the wavelength are both increased.

Refraction of Light Waves

The above discussion was limited to the behavior of a wave on a rope. But what if the wave is
a light wave traveling in a three-dimensional medium? For example, what would happen if a
light wave is traveling through air and reaches the boundary with a glass surface? How can the
reflection and transmission behavior of a light wave be described? First, the light wave
behaves like the wave on the rope: a portion of the wave is transmitted into the new medium
(glass) and a portion of the wave reflects off the air-glass boundary. Second, the same wave
property changes which were observed for the wave on the rope are also observed for the
light wave passing from air into glass; there is a change in speed and wavelength of the wave
as it crosses the air-glass boundary. When passing from air into glass, both the speed and the
wavelength decrease. Finally, and most importantly, the light is observed to change directions
as it crosses the boundary separating the air and the glass. This bending of the path of light is
known as refraction. A one-word synonym for refraction is bending. The transmitted wave
experiences this refraction at the boundary. As seen in the diagram at the right, each
individual wavefront is bent only along the boundary. Once the wavefront has passed across
the boundary, it travels in a straight line. For this reason, refraction is called a boundary
behavior. A ray is drawn perpendicular to the wavefronts; this ray represents the direction
which the light wave is traveling. Observe that the ray is a straight line inside of each of the
two media, but bends at the boundary. Again, refraction is a boundary behavior.

The Ray Model of Light

In this unit, we will rely heavily on the use of rays to represent the direction in which light is
moving. While we often think of light behaving as a wave, we will still find it useful to
represent its movement through a medium using a line segment with an arrowhead (i.e., a
ray) to depict the refraction of light. The ray is constructed in a direction perpendicular to the
wavefronts of the light wave; this accurately depicts the light wave's direction. In this sense,
we are viewing light as behaving as a stream of particles which head in the direction of the
ray. The idea that the path of light can be represented by a ray is known as the ray model of
light. The same ray model was utilized in Unit 13 of The Physics Classroom Tutorial to discuss
the reflection of light waves.

Refraction at a Boundary
Refraction and Sight
In Unit 13 of The Physics Classroom Tutorial, it was emphasized that we are able to see
because light from an object can travel to our eyes. Every object that can be seen is seen only
because light from that object travels to our eyes. As you look at Mary in class, you are able to
see Mary because she is illuminated with light and that light reflects off of her and travels to
your eye. In the process of viewing Mary, you are directing your sight along a line in the
direction of Mary. If you wish to view the top of Mary's head, then you direct your sight along
a line towards the top of her head. If you wish to view Mary's feet, then you direct your sight
along a line towards Mary's feet. And if you wish to view the image of Mary in a mirror, then
you must direct your sight along a line towards the location of Mary's image. This directing of
our sight in a specific direction is sometimes referred to as the line of sight.

As light travels through a given medium, it travels in a straight line. However, when light
passes from one medium into a second medium, the light path bends. Refraction takes place.
The refraction occurs only at the boundary. Once the light has crossed the boundary between
the two media, it continues to travel in a straight line. Only now, the direction of that line is
different than it was in the former medium. If when sighting at an object, light from that
object changes media on the way to your eye, a visual distortion is likely to occur. This visual
distortion is witnessed if you look at a pencil submerged in a glass half-filled with water. As
you sight through the side of the glass at the portion of the pencil located above the water's
surface, light travels directly from the pencil to your eye. Since this light does not change
medium, it will not refract. (Actually, there is a change of medium from air to glass and back
into air. Because the glass is so thin and because the light starts and finished in air, the
refraction into and out of the glass causes little deviation in the light's original direction.) As
you sight at the portion of the pencil which was submerged in the water, light travels from
water to air (or from water to glass to air). This light ray changes medium and subsequently
undergoes refraction. As a result, the image of the pencil appears to be broken. Furthermore,
the portion of the pencil which is submerged in water appears to be wider than the portion of
the pencil which is not submerged. These visual distortions are explained by the
refraction of light.

In this case, the light rays which undergo a deviation from their original path are
those which travel from the submerged portion of the pencil, through the water,
across the boundary, into the air, and ultimately to the eye. At the boundary, this ray refracts.
The eye-brain interaction cannot account for the refraction of light. As was emphasized in Unit
13, the brain judges the image location to be the location where light rays appear to originate
from. This image location is the location where either reflected or refracted rays intersect. The
eye and brain assume that light travels in a straight line and then extends all incoming rays of
light backwards until they intersect. Light rays from the submerged portion of the pencil will
intersect in a different location than light rays from the portion of the pencil which extends
above the surface of the water. For this reason, the submerged portion of the pencil appears to
be in a different location than the portion of the pencil which extends above the water. The
diagram at the right shows a God's-eye view of the light path from the submerged portion of
the pencil to each of your two eyes. Only the left and right extremities (edges) of the pencil
are considered. The blue lines depict the path of light to your right eye and the red lines depict
the path of light to your left eye. Observe that the light path has bent at the boundary. Dashed
lines represent the extensions of the lines of sight backwards into the water. Observe that the
these extension lines intersect at a given point; the point represents the image of the left and
the right edge of the pencil. Finally, observe that the image of the pencil is wider than the
actual pencil. A ray model of light which considers the refraction of light at boundaries
adequately explains the broken pencil observations.

The broken pencil phenomenon occurs during your everyday spear-fishing outing. Fortunately
for the fish, light refracts as it travels from the fish in the water to the eyes of the hunter. The
refraction occurs at the water-air boundary. Due to this bending of the path of light, a fish
appears to be at a location where it isn't. A visual distortion occurs. Subsequently, the hunter
launches the spear at the location where the fish is thought to be and misses the fish. Of
course, the fish are never concerned about such hunters; they know that light refracts at the
boundary and that the location where the hunter is sighting is not the same location as the
actual fish. How did the fish get so smart and learn all this? They live in schools.
Now any fish who has done his/her physics homework knows that the amount of refraction
which occurs is dependent upon the angle at which the light approaches the boundary. We will
investigate this aspect of refraction in great detail in Lesson 2. For now, it is sufficient to say
that as the hunter with the spear sights more perpendicular to the water, the amount of
refraction decreases. The most successful hunters are those who sight perpendicular to the
water. And the smartest fish are those who head for the deep when they spot hunters who
sight in this direction.

Since refraction of light occurs when it crosses the boundary, visual distortions often occur.
These distortions occur when light changes medium as it travels from the object to our eyes.

Refraction at a Boundary
The Cause of Refraction
We have learned that refraction occurs as light passes across the boundary between two
medium. Refraction is merely one of several possible boundary behaviors by which a light
wave could behave when it encounters a new medium or an obstacle in its path. The
transmission of light across a boundary between two media is accompanied by a change in
both the speed and wavelength of the wave. The light wave not only changes directions at the
boundary, it also speeds up or slows down and transforms into a wave with a larger or a
shorter wavelength. The only time that a wave can be transmitted across a boundary, change
its speed, and still not refract is when the light wave approaches the boundary in a direction
which is perpendicular to it. As long as the light wave changes speed and approaches the
boundary at an angle, refraction is observed.

But why does light refract? What is the cause of such behavior? And why is there this one
exception to the refraction of light? An analogy of marching soldiers is often used to address
this question. In fact, it is not uncommon that the analogy be illustrated in a Physics class with
a student demonstration. A group of students forms a straight line (shoulder to shoulder)
and connect themselves to their nearest neighbor using meter sticks. A strip of masking tape
divides the room into two media. In one of the media (on one side of the tape), students walk
at a normal pace. In the other media (or on the other side of the tape), students walk very
slowly using baby steps. The group of students walk forward together in a straight line
towards the diagonal strip of masking tape. The students maintain the line as they approach
the masking tape. When an individual student reaches the tape, that student abruptly changes
the pace of her/his walk. The group of students continue walking until all students in the line
have entered into the second medium. The diagram below represents the line of students
approaching the boundary (the masking tape) between the two medium. On the diagram, an
arrow is used to show the general direction of travel for the group of students in both medium.
Observe that the direction of the students changes at the "boundary."

The fundamental feature of the students' motion which leads to this change in direction is the
change in speed. Upon reaching the masking tape, each individual student abruptly changes
speed. Because the students approach the masking tape at an angle, each individual student
reaches the tape at a different time. The student who reaches the tape first, slows down while
the rest of the line of students marches ahead. This occurs for every student in the line of
students. Once a student reaches the boundary, that student slows down while his/her nearest
neighbor marches ahead at the original pace. The result is that the direction that the line of
students is heading is altered at the boundary. The change in speed of the line of students
causes a change in direction.

Conditions of Refraction

Will this refractive behavior always occur? No! There are two conditions which are required in
order to observe the change in direction of the path of the students:

• The students must change speed when crossing the boundary.


• The students must approach the boundary at an angle; refraction will not occur
when they approach the boundary head-on (i.e., heading perpendicular to it).
These are both reasonable enough conditions if you consider the previous paragraph. If the
students do not change speed, then there is no cause factor. Recall that it was the change in
speed of the students which caused the change in direction. The second condition is also
reasonable. If the students approach the masking tape in a direction which is perpendicular to
it , then each student will reach the tape at the exact same time. Recall that the line of
student changed their direction because they had reached the masking tape at different times.
The first student reached the tape, slowed down, and observed the rest of the students
marching ahead at the original speed. The change in direction of the line of students only
occurs at the boundary when the students change speed and approach at an angle.

The Marching Soldiers analogy provides an excellent analogy to understanding the cause of
light refraction. The line of students approaching the masking tape are analogous to a
wavefront of light. The masking tape is analogous to a boundary between two medium. The
change in speed which occurred for the line of students would also occur for a wave of light.
And like the marching students, a light wave will not undergo refraction if it approaches the
boundary in a direction which is perpendicular to it.

The same two conditions which are necessary for bending the path of the line of students are
also necessary for bending the direction of a light ray. Light refracts at a boundary because of
a change in speed. Their is a distinct cause-effect relationship. The change in speed is the
cause and the change in direction (refraction) is the effect.

Refraction at a Boundary
Optical Density and Light Speed
Refraction is the bending of the path of a light wave as it passes from one material to another
material. The refraction occurs at the boundary and is caused by a change in the speed of the
light wave upon crossing the boundary. The tendency of a ray of light to bend one direction or
another is dependent upon whether the light wave speeds up or slows down upon crossing the
boundary. Because a major focus of our study will be upon the direction of bending, it will be
important to understand the factors which effect the speed at which a light wave is
transported through a medium.

The mechanism by which a light wave is transported through a medium occurs in a manner
which is similar to the way that any other wave is transported - by particle-to-particle
interaction. In Unit 10 of The Physics Classroom Tutorial, the particle-to-particle interaction
mechanism by which a mechanical wave transports energy was discussed in detail. In Unit 12
of The Physics Classroom Tutorial, the mechanism of energy transport by an electromagnetic
wave was briefly discussed. Here we will look at this method in more detail.

An electromagnetic wave (i.e., a light wave) is produced by a vibrating electric charge. As the
wave moves through the vacuum of empty space, it travels at a speed of c (3 x 108m/s). This
value is the speed of light in a vacuum. When the wave impinges upon a particle of matter, the
energy is absorbed and sets electrons within the atoms into vibrational motion. If the
frequency of the electromagnetic wave does not match theresonant frequency of vibration of
the electron, then the energy is reemitted in the form of an electromagnetic wave. This new
electromagnetic wave has the same frequency as the original wave and it too will travel at a
speed of c through the empty space between atoms. The newly emitted light wave continues
to move through the interatomic space until it impinges upon a neighboring particle. The
energy is absorbed by this new particle and sets the electrons of its atoms into vibration
motion. And once more, if there is no match between the frequency of the electromagnetic
wave and the resonant frequency of the electron, the energy is reemitted in the form of a new
electromagnetic wave. The cycle of absorption and reemission continues as the energy is
transported from particle to particle through the bulk of a medium. Every photon (bundle of
electromagnetic energy) travels between the interatomic void at a speed of c; yet time delay
involved in the process of being absorbed and reemitted by the atoms of the material lowers
the net speed of transport from one end of the medium to the other. Subsequently, the net
speed of an electromagnetic wave in any medium is somewhat less than its speed in a vacuum
- c (3 x 108 m/s).

Optical Density and the Index of Refraction

Like any wave, the speed of a light wave is dependent upon the properties of the medium. In
the case of an electromagnetic wave, the speed of the wave depends upon the optical
density of that material. The optical density of a medium is not the same as its physical
density. The physical density of a material refers to the mass/volume ratio. The optical density
of a material relates to the sluggish tendency of the atoms of a material to maintain the
absorbed energy of an electromagnetic wave in the form of vibrating electrons before
reemitting it as a new electromagnetic disturbance. The more optically dense which a material
is, the slower that a wave will move through the material.

One indicator of the optical density of a material is the index of refraction value of the
material. Index of refraction values (represented by the symbol n) are numerical index values
which are expressed relative to the speed of light in a vacuum. The index of refraction value of
a material is a number which indicates the number of times slower that a light wave would be
in that material than it is in a vacuum. A vacuum is given an n value of 1.0000. The n values
of other materials are found from the following equation:

The table below lists index of refraction values for a variety of medium. The materials listed at
the top of the table are those through which light travels fastest; these are the least optically
dense materials. The materials listed at the bottom of the table are those through which light
travels slowest; these are the most optically dense materials. So as the index of refraction
value increases, the optical density increases, and the speed of light in that material
decreases.

Material Index of Refraction


Vacuum 1.0000
<--lowest optical density

Air 1.0003
Ice 1.31
Water 1.333
Ethyl Alcohol 1.36
Plexiglas 1.51
Crown Glass 1.52
Light Flint Glass 1.58
Dense Flint Glass 1.66
Zircon 1.923
Diamond 2.417
Rutile 2.907
Gallium phosphide 3.50
<--highest optical density

The index of refraction values thus provide a measure of the relative speed of a light wave in a
particular medium. Knowledge of such relative speeds allows a student of physics to predict
which way a light ray would bend when passing from one medium to the another. In the next
part of Lesson 1, the rules for the direction of bending will be discussed in detail.

Refraction at a Boundary
The Direction of Bending
Refraction is the bending of the path of a light wave as it passes from one material into
another material. The refraction occurs at the boundary and is caused by a change in the
speed of the light wave upon crossing the boundary. The tendency of a ray of light to bend one
direction or another is dependent upon whether the light wave speeds up or slows down upon
crossing the boundary. The speed of a light wave is dependent upon the optical density of the
material through which it moves. For this reason, the direction which the path of a light wave
bends depends on whether the light wave is traveling from a more dense (slow) medium to a
less dense (fast) medium or from a less dense medium to a more dense medium. In this part
of Lesson 1, we will investigate this topic of the direction of bending of a light wave.

Recall the Marching Soldiers analogy discussed earlier in this lesson. The analogy served as a
model for understanding the boundary behavior of light waves. As discussed, the analogy is
often illustrated in a Physics classroom by a student demonstration. In the demonstration, a
line of students (representing a light wave) march towards a masking tape (representing the
boundary) and slow down upon crossing the boundary (representative of entering a new
medium). The direction of the line of students changes upon crossing the boundary. The
diagram below depicts this change in direction for a line of students who slow down upon
crossing the boundary.

On the diagram, the direction of the students is represented by two arrows known asrays. The
direction of the students as they approach the boundary is represented by anincident
ray (drawn in blue). And the direction of the students after they cross the boundary is
represented by a refracted ray (drawn in red). Since the students change direction (i.e.,
refract), the incident ray and the refracted ray do not point in the same direction. Also, note
that a perpendicular line is drawn to the boundary at the point where the incident ray strikes
the boundary (i.e., masking tape). A line drawn perpendicular to the boundary at the point of
incidence is known as a normal line. Observe that the refracted ray lies closer to the normal
line than the incident ray does. In such an instance as this, we would say that the path of the
students has benttowards the normal. We can extend this analogy to light and conclude
that:

Light Traveling from a Fast to a Slow Medium


If a ray of light passes across the boundary from a material in which it travels fast into a material in which
travels slower, then the light ray will bend towards the normal line.

The above principle applies to light passing from a material in which it travels fast across a
boundary and into a material in which it travels slow. But what if light wave does the opposite?
What if a light wave passes from a material in which it travels slow across a boundary and into
a material in which it travels fast? The answer to this question can be answered if we
reconsider the Marching Soldier analogy. Now suppose that the each individual student in the
train of students speeds up once they cross the masking tape. The first student to reach the
boundary will speed up and pull ahead of the other students. When the second student
reaches the boundary, he/she will also speed up and pull ahead of the other students who
have not yet reached the boundary. This continues for each consecutive student, causing the
line of students to now be traveling in a direction further from the normal. This is depicted in
the diagram below.
From this analogy and the diagram above, we see that the refracted ray (in red)
is further away from the normal then the incident ray (in blue). In such an
instance as this, we would say that the path of the students has bent away
from the normal. We can once more extend this analogy to light and conclude that:

Light Traveling from a Slow to a Fast Medium


If a ray of light passes across the boundary from a material in which it travels slow into a material in which
travels faster, then the light ray will bend away from the normal line.

The Tractor Analogy

Now lets consider another analogy to assist in our understanding of these two important
principles. Suppose that a tractor is moving across an asphalt surface towards a rectangular
plot of grass (as shown in the diagram at the right). Upon entering the grass, the tractors'
wheels will sink into the surface and slow down. Upon exiting the plot of grass on the opposite
side, the tractor wheels will speed up and achieve their original speed. In effect, this analogy
would be representative of a light wave crossing two boundaries. At the first boundary (the
asphalt to grass boundary), the light wave (or the tractor) would be slowing down; and at the
second boundary (the grass to asphalt boundary), the light wave (or the tractor) would be
speeding up. We can apply our two important principles listed above and predict the direction
of bending and the path of the tractor as it travels through the rectangular plot of grass. As
indicated on the diagram, upon entering the grass, the wheels slow down and the path of the
tractor bends towards the normal (perpendicular line drawn to the surface). Upon exiting the
plot of grass, the wheels speed up and the path of the tractor bends away from the normal.
The path of the tractor is closer to the normal in the slower medium and farther away from the
normal in the faster medium.

This analogy can be extended to the path of a light wave as it passes from air into and out of a
rectangular block of glass. Since air is less optically dense than glass, the light wave will slow
down upon entering the glass and speed up when exiting the glass. In other words, the light
wave will be undergoing the same change in speed as the tractor in the above diagram. For
this reason, the direction of bending for the light wave upon entering and exiting the glass will
be the same as in the diagram above. The light ray refracts towards the normal upon entering
the glass (crossing from a fast to a slow medium) and refracts away from the normal upon
exiting the glass (crossing from a slow to a fast medium). This is shown in the diagram at the
right.

There is an important point to be noted in these diagrams of the rectangular plot of grass and
rectangular block of glass. Notice that the direction of the original incident ray is the same as
the direction of the final refracted ray. Put another way, the direction at which the light is
traveling when entering the rectangular block of glass is the same as the direction which the
light travels after exiting the rectangular block of glass. There is no ultimate change in the
direction which the light is traveling. This small detail will only be the case under two
conditions:

• the two sides of the glass through which the light enters and exits are parallel
to each other
• the medium surrounding the glass on the side through which the light enters
and exits are the same

These two conditions are met in the case of a rectangular block of glass surrounded by air.

The diagrams below provides a contrast to the rectangular plot of grass and the rectangular
block of glass. Both diagrams involve the refraction of a tractor or a light wave as it passes
into and out of a triangular plot of grass and a triangular block of glass.

Copy this diagram onto a sheet of paper and apply your understanding of refraction principles
to predict the path of the tractor and the light wave as it travels through the triangle-shaped
obstacle. Draw the path on your separate sheet of paper and then click on the button below to
check your answer
Least Time Principle

Another means of approaching the subject of the direction which light bends when crossing a
boundary between two medium is through the Least Time Principle. This Least Time
Principle is sometimes stated as follows:

Least Time Principle


Of all the possible paths that light might take to get from one point to another, it always takes the path that
requires the least amount of time.

A useful analogy to understanding the principle involves a life guard who has become aware of
a drowning swimmer in the water. In order to save the drowning swimmer, the life guard must
run through the sand, cross the boundary between the sand and the water, and then swim
through the water to the drowning swimmer. Of course, the guard must reach the swimmer in
as little time as possible. Since the guard can run faster on sand than she can swim in water, it
would make sense that the guard cover more distance in the sand than she does in the water.
In other words, she will not run directly at the drowning swimmer. The optimal entry point into
the water is the point which would allow the life guard to reach the drowning swimmer in the
least amount of time. Obviously, this point would be at a location closer to the swimmer than
to the guard. The diagram below depicts such an entry point.

Observe in the diagram, that minimizing the time to reach the swimmer means that the life
guard will approach the boundary at a steep angle to the normal and then will bend towards
the normal upon crossing the boundary. This analogy demonstrates that the Least Time
Principle would predict the following direction of bending:

A ray of light will bend towards the normal when crossing the boundary from a medium in
which it travels fast into a medium in which it travels slow.
This is the very generalization which was made earlier on this page.
Some Useful Mnemonics

Using the above principles and logic to explain and predict the direction which light refracts
when crossing a boundary will be a major objective of this unit. Rather than merely restating
the principle, you will be asked to apply it to a variety of situations (such as those in the Check
Your Understanding section below). Part of accomplishing this task will involve remembering
the principles. For this reason, the following useful mnemonics are offered.

FST = Fast to Slow, Towards Normal


If a ray of light passes across the boundary from a material in which it travelsfast into a material in which
travels slower, then the light ray will bend towards the normal line.
SFA = Slow to Fast, Away From Normal
If a ray of light passes across the boundary from a material in which it travelsslow into a material in which
travels faster, then the light ray will bend away from the normal line.

A mnemonic is a tool used to help one remember and difficult-to-remember idea. Of course,
there is always the risk that the mnemonic will be forgotten. And since FST and SFA might not
be the most easily remembered mnemonics, perhaps the following oddity will help. You can
remember FST (fast to slow; towards) by simply thinking about thoseFreaky Science
Teachers which you have had through the years. And you can remember SFA by thinking
about the disgusting habit of your friend Sara (or Susan or Sammy or Samir or Somebody ...)
- Sarah Farts Alot.

Check Your Understanding

Test your ability to apply these principles by answering the following questions.

1. When light passes from a more optically dense medium into a less optically densemedium,
it will bend _______ (towards, away from) the normal.

2. When light passes from a less optically dense medium into a more optically densemedium,
it will bend _______ (towards, away from) the normal.
3. When light passes from a medium with a high index of refraction value into a medium with
a low index of refraction value, it will bend _______ (towards, away from) the normal.

4. When light passes from a medium with a low index of refraction value into a medium with a
high index of refraction value, it will bend _______ (towards, away from) the normal.

5. In each diagram, draw the "missing" ray (either incident or refracted) in order to
appropriately show that the direction of bending is towards or away from the normal.

6. Arthur Podd's method of fishing involves spearing the fish while standing on the shore. The
actual location of a fish is shown in the diagram below. Because of the refraction of light, the
observed location of the fish is different than its actual location. Indicate on the diagram the
approximate location where Arthur observes the fish to be. Must Arthur aim above or below
where the fish appears to be in order to strike the fish?
7. For the following two cases, state whether the ray of light will bend towards or away from
the normal upon crossing the boundary.

Refraction at a Boundary
If I Were an Archer Fish
In the quiet waters of the Orient, there is an unusual fish known as the Archer fish. The Archer
fish is unlike any other fish in that the Archer fish finds its prey living outside the water. An
insect, butterfly, spider or similar creature is the target of the Archer fish's powerful spray of
water. The Archer fish will search for prey that are resting upon a branch or twig above the
water. With pinpoint accuracy, the fish knocks the prey off the branch using a powerful jet of
water. The prey falls to the water as the Archer fish simultaneously swims directly to the
location on the surface where the prey strikes the water, wasting no time to retrieve its meal.

The feat of shooting a stream of water to knock the prey off a branch is remarkable. The fact
that the Archer fish can do this time and again with pinpoint accuracy is even more
remarkable. The fact that the fish can determine the exact location of incidence at which the
prey will subsequently strike the water is incredibly remarkable. But most remarkable of all is
that the Archer fish can accomplish this trick despite the fact that light from the prey to
its eye undergoes refraction at the air-water boundary. Such refraction would cause
avisual distortion, making the prey appear to be in a location where it isn't. Yet the
Archer fish is hardly ever fooled. What is the secret of the Archer fish? How is it able to
overcome the visual distortion caused by refraction in order to accomplish these
remarkable hunting tasks? Biologists are not quite sure, though as of this writing it is a topic
which is under considerable experimental investigation.

Now we will entertain the question: What if I were an Archer fish? What could I do to perform
such a remarkable aquatic magic trick? Unlike a fish, I don't live in schools; nonetheless, there
is a principle taught in schools that might assist me in such a trick. That principle is usually
taught in a Physics classroom.

There is only one condition in which light can pass from one medium to another, change its
speed, and still not refract. If the light is traveling in a direction which is perpendicular to the
boundary, no refraction occurs. As the light wave crosses over the boundary, its speed and
wavelength still change. Yet, since the light wave is approaching the boundary in a
perpendicular direction, each point on the wavefront will reach the boundary at the same time.
For this reason, there is no refraction of the light. Such a ray of light is said to be approaching
the boundary while traveling along the normal. (The normal is a line drawn perpendicular to
the surface.)

The only means by which I could remotely match the marvels of the Archer fish would be to
line up my sight with the prey from a position directly underneath the prey. From this vantage
point, light from the prey travels directly to my eye without undergoing a change in direction.
Since the light is traveling along the normal to the surface, it does not refract. The light passes
straight through the water to my eyes. Normally, when light from an object changes medium
on the way to the eye, there is a visual distortion of the image. But if I sight along the normal,
there is no refraction and no visual distortion of the image. From this ideal line of sight, I
would be able to hit my prey time after time (assuming I could master the task of spraying a
jet of water in the desired direction). Using my physics understanding of the refraction of light,
I could pretend to mimic the Archer fish.
And now for the rest of the story. From this discussion, one might conclude that the secret of
the Archer fish is to aim at its prey from directly below. Refraction is less when sighting along
the normal. However, the Archer fish's accomplishments are more remarkable than that. It has
been found that Archer fish are able to strike their prey when sighting upwards at angles of 40
degrees with the normal. In fact, it has been found that hit probabilities do not show
significant variance with the angle of sighting, meaning that an Archer fish is just as likely to
strike its prey whether the amount of refraction is great or minimal. Now that's remarkable!

The Mathematics of Refraction


The Angle of Refraction
Refraction is the bending of the path of a light wave as it passes across the boundary
separating two media. Refraction is caused by the change in speed experienced by a wave
when it changes medium. In Lesson 1, we learned that if a light wave passes from a medium
in which it travels slow (relatively speaking) into a medium in which it travels fast, then the
light wave will refract away from the normal. In such a case, the refracted ray will be farther
from the normal line than the incident ray; this is the SFA rule of refraction. On the other
hand, if a light wave passes from a medium in which it travels fast (relatively speaking) into a
medium in which it travels slow, then the light wave will refract towards the normal. In such a
case, the refracted ray will be closer to the normal line than the incident ray is; this is the FST
rule of refraction. These two rules regarding the refraction of light only indicate the direction
which a light ray bends; they do not indicate how much bending occurs. Lesson 1 focused on
the topics of "What causes refraction?" and "Which direction does light refract?" Lesson 2 will
focus on the question of "By how much does light refract when it crosses a boundary?"
The question is: "By how much does light refract when it crosses a boundary?"
Perhaps there are numerous answers to such a question. (For example, " a lot,"
"a little," "like wow! quite a bit dude," etc.) The concern of this lesson is to
express the amount of refraction of a light ray in terms of a measurable
quantity that has a mathematical value. The diagram to the right shows a light
ray undergoing refraction as it passes from air into water. As mentioned in Lesson 1,
the incident rayis a ray (drawn perpendicular to the wavefronts) which shows the direction
which light travels as it approaches the boundary. (The meaning of an incident ray was first
introduced in the discussion of Reflection of Light in Unit 13 of The Physics Classroom
Tutorial.) Similarly, the refracted ray is a ray (drawn perpendicular to the wavefronts) which
shows the direction which light travels after it has crossed over the boundary. In the diagram,
a normal line is drawn to the surface at the point of incidence, This line is always drawn
perpendicular to the boundary. The angle which the incident ray makes with the normal line is
referred to as the angle of incidence. Similarly, the angle which the refracted ray makes with
the normal line is referred to as the angle of refraction. The angle of incidence and angle of
refraction are denoted by the following symbols:

= angle of incidence

= angle of refraction

The amount of bending which a light ray experiences can be expressed in terms of the angle
of refraction (more accurately, by the difference between the angle of refraction and the angle
of incidence). A ray of light may approach the boundary at an angle of incidence of 45-degrees
and bend towards the normal. If the medium into which it enters causes a small amount of
refraction, then the angle of refraction might be a value of about 42-degrees. On the other
hand if the medium into which the light enters causes a large amount of refraction, the angle
of refraction might be 22-degrees. (These values are merely arbitrarily chosen values to
illustrate a point.) The diagram below depicts a ray of light approaching three different
boundaries at an angle of incidence of 45-degrees. The refractive medium is different in each
case, causing different amounts of refraction. The angles of refraction are shown on the
diagram.
Of the three boundaries in the diagram, the light ray refracts the most at the air-
diamond boundary. This is evident by the fact that the difference between the angle of
incidence and the angle of refraction is greatest for the air-diamond boundary. But how
can this be explained? The cause of refraction is a change in light speed; and wherever
the light speed changes most, the refraction is greatest. We have already learned that the
speed is related to the optical density of a material which is related to the index of
refraction of a material. Of the four materials present in the above diagram, air is the least
dense material (lowest index of refraction value) and diamond is the most dense material
(largest index of refraction value). Thus, it would be reasonable that the most refraction
occurs for the transmission of light across an air-diamond boundary.

In this example, the angle of refraction is the measurable quantity which indicates the amount
of refraction taking place at any boundary. A comparison of the angle of refraction to the angle
of incidence provides a good measure of the refractive ability of any given boundary. For any
given angle of incidence, the angle of refraction is dependent upon the speeds of light in each
of the two materials. The speed is in turn dependent upon the optical density and the index of
refraction values of the two materials. There is a mathematical equation relating the angles
which the light rays make with the normal to the indices (plural for index) of refraction of the
two materials on each side of the boundary. This mathematical equation is known as Snell's
Law and is the topic of the next section of Lesson 2.

The Mathematics of Refraction


Snell's Law
Refraction is the bending of the path of a light wave as it passes across the boundary
separating two media. Refraction is caused by the change in speed experienced by a wave
when it changes medium. Lesson 1, focused on the topics of "What causes refraction?" and
"Which direction does light refract?" In that lesson, we learned that light can either
refract towards the normal (when slowing down while crossing the boundary) or away from
the normal (when speeding up while crossing the boundary). The focus of Lesson 2 is upon the
question of "By how much does light refract when it crosses a boundary?" In the first part of
Lesson 2, we learned that a comparison of the angle of refraction to the angle of incidence
provides a good measure of the refractive ability of any given boundary. The more that light
refracts, the bigger the difference between these two angles. In this part of Lesson 2, we will
learn about a mathematical equation relating these two angles and the indices of refraction of
the two materials on each side of the boundary.

To begin, consider a hemi-cylindrical dish filled with water. Suppose that a laser beam is
directed towards the flat side of the dish at the exact center of the dish. The angle of incidence
can be measured at the point of incidence. This ray will refract, bending towards the normal
(since the light is passing from a medium in which it travels fast into one in which it travels
slow - FST). Once the light ray enters the water, it travels in a straight line until it reaches the
second boundary. At the second boundary, the light ray is approaching along the normal to the
curved surface (this stems from the geometry of circles). The ray does not refract upon exiting
since the angle of incidence is 0-degrees (recall the If I Were An Archer Fishpage). The ray of
laser light therefore exits at the same angle as the refracted ray of light made at the first
boundary. These two angles can be measured and recorded. The angle of incidence of the
laser beam can be changed to 5-degrees and new measurements can be made and recorded.
This process can be repeated until a complete data set of accurate values has been collected.
The data below show a representative set of data for such an experiment.

Angle of Incidence (degrees) Angle of Refraction (degrees)


0.00 0.00
5.00 3.8
10.0 7.5
15.0 11.2
20.0 14.9
25.0 18.5
30.0 22.1
35.0 25.5
40.0 28.9
45.0 32.1
50.0 35.2
55.0 38.0
60.0 40.6
65.0 43.0
70.0 45.0
75.0 46.6
80.0 47.8
85.0 48.5

An inspection of the data above reveal that there is no clear linear relationship between the
angle of incidence and the angle of refraction. For example, a doubling of the angle of
incidence from 40 degrees to 80 degrees does not result in a doubling of the angle of
refraction. Thus, a plot of this data would not yield a straight line. If however, the sine of the
angle of incidence and the sine of the angle of refraction were plotted, the plot would be a
straight line, indicating a linear relationship between the sines of the important angles. If two
quantities form a straight line on a graph, then a mathematical relationship can be written in y
= m*x + b form. A plot of the sine of the angle of incidence vs. the sine of the angle of
refraction is shown below.
The equation relating the angles of incidence ("theta i") and the angle of refraction ("theta r")
for light passing from air into water is given as

Observe that the constant of proportionality in this equation is 1.33 - the index of refraction
value of water. Perhaps it's just a coincidence. But if the semi-cylindrical dish full of water was
replaced by a semi-cylindrical disk of Plexiglas, the constant of proportionality would be 1.51
- the index of refraction value of Plexiglas. This is not just a coincidence. The same pattern
would result for light traveling from air into any material. Experimentally, it is found that for a
ray of light traveling from air into some material, the following equation can be written.

where nmaterial = index of refraction of the material

This study of the refraction of light as it crosses from one material into a second material
yields a general relationship between the sines of the angle of incidence and the angle of
refraction. This general relationship is expressed by the following equation:

where ("theta i") = angle of incidence

("theta r") = angle of refraction

ni = index of refraction of the incident medium

nr = index of refraction of the refractive medium

This relationship between the angles of incidence and refraction and the indices of refraction of
the two medium is known as Snell's Law. Snell's law applies to the refraction of light in any
situation, regardless of what the two media are.
Using Snell's Law to Predict An Angle Value

As with any equation in physics, the Snell's Law equation is valued for its predictive ability. If
any three of the four variables in the equation are known, the fourth variable can be predicted
if appropriate problem-solving skills are employed. This is illustrated in the two examples
below.

Example Problems
In the following two examples, use Snell's law, the sine button on your calculator, a protractor, and the index of
refraction values to complete the following diagrams. Measure , calculate , and draw in the refracted ray with the
calculated angle of refraction.

In each of these two example problems, the angle of refraction is the variable to be
determined. The indices of refraction (ni and nr) are given and the angle of incidence can be
measured. With three of the four variable known, substitution into Snell's law followed by
algebraic manipulation will lead to the answer.

Solution to Example A
First, use a protractor to measure the angle of incidence. An appropriate measurement would
be some angle close to 45-degrees.

Second, list all known values and the unknown value for which you wish to solve:

Given: Find:

ni = 1.00 nr = 1.33 = 45 degrees = ???

Third, list the relevant equation:


Fourth, substitute known values into the equation and algebraically manipulate the equation in
order to solve for the unknown variable - "theta r."

1.00 * sine (45 degrees) = 1.33 * sine (theta r)


0.7071 = 1.33 * sine (theta r)

0.532 = sine (theta r)

sine-1 (0.532) = sine-1 ( sine (theta r))

32.1 degrees = theta r

Proper algebra yields to the answer of 32.1 degrees for the angle of refraction. The diagram
showing the refracted ray can be viewed by clicking the button below.

The solution to Example A is given as an example. Try Example B on your own and click on the
button to check your answer.

Snell's Law provides the quantitative means of answering the question of "By how much does
the light ray refract?" The task of answering this question involves using indices of refraction
and the angle of incidence values in order to determine the angle of refraction. This problem-
solving process is discussed in more detail on the remaining pages of Lesson 2.

Next Section: Ray Tracing and Problem-Solving


Jump To Lesson 3: Total Internal Reflection
The Mathematics of Refraction
Ray Tracing and Problem-Solving
In a previous part of Lesson 2, we learned about a mathematical equation relating the two
angles (angles of incidence and refraction) and the indices of refraction of the two materials on
each side of the boundary. The equation is known as the Snell's Law equation and is expressed
as follows.

where ("theta i") = angle of incidence

("theta r") = angle of refraction

ni = index of refraction of the incident medium

nr = index of refraction of the refractive medium

As with any equation in physics, the Snell's Law equation is valued for its predictive ability. If
any three of the four variables in the equation are known, the fourth variable can be predicted
if appropriate problem-solving skills are employed. In this part of Lesson 2, we will investigate
several of the types of problems that you will have to solve, and learn the task of tracing the
refracted ray if given the incident ray and the indices of refraction.

Example Problem A
A ray of light in air is approaching the boundary with water at an angle of 52 degrees. Determine the angle of refraction
of the light ray. Refer to the table of indices of refraction if necessary.

Solution to Problem A
The solution to this problem begins like any problem: a diagram is constructed to assist in the
visualization of the physical situation, the known values are listed, and the unknown value
(desired quantity) is identified. This is shown below:

Diagram: Given: Find:

ni = 1.00 (from table) =??


nr = 1.333 (from table)

= 52 degrees

Now list the relevant equation (Snell's Law), substitute known values into the equation, and
perform the proper algebraic steps to solve for the unknown.

1.00 * sine (52 degrees) = 1.333 * sine (theta r)

0.7880 = 1.333 * sine (theta r)


0.591 = sine (theta r)

sine-1 (0.591) = sine-1 ( sine (theta r))

36.2 degrees = theta r

Proper algebra yields the answer of 36.2 degrees for the angle of refraction. When finished, it
is always a wise idea to apply the FST and SFA principles as a check of your numerical answer.
In this problem, the light ray is traveling from a less optically dense or fast medium (air) into
a more optically dense or slow medium (water), and so the light ray should refract towards the
normal - FST. Thus, the angle of refraction should be smaller than the angle of refraction. And
indeed it is - 36.2 degrees (theta r) is smaller than 52.0 degrees (theta i). Using this
conceptual criteria as a check of your answer can often identify incorrect solutions to
problems.

Example Problem B
A ray of light in air is approaching the boundary with a layer of crown glass at an angle of 42.0 degrees. Determine the
angle of refraction of the light ray upon entering the crown glass and upon leaving the crown glass. Refer to the table of
indices of refraction if necessary.
Solution to Problem B
This problem is slightly more complicated than Problem A since refraction is taking place at
two boundaries. This is an example of a layer problem where the light refracts upon entering
the layer (boundary #1: air to crown glass) and again upon leaving the layer (boundary #2:
crown glass to air). Despite this complication, the solution begins like the above problem: a
diagram is constructed to assist in the visualization of the physical situation, the known values
are listed, and the unknown value (desired quantity) is identified. This is shown below:

Diagram: Given: Find:


Boundary #1 at

ni = 1.00 (from table) boundary #1

nr = 1.52 (from table) and

= 42.0 degrees at
Note that the angle of
boundary #2
refraction at boundary #1
Boundary #2
is the same as the angle of
incidence at boundary #2. ni = 1.52 (from table)

nr = 1.00 (from table)

Now list the relevant equation (Snell's Law), substitute known values into the equation, and
perform the proper algebraic steps to solve for the unknown. Begin the process at boundary
#1 and then repeat for boundary #2 until the final answer is found.

Boundary #1:

1.00 * sine (42.0 degrees) = 1.52 * sine (theta r)

0.669 = 1.52 * sine (theta r)

0.4402 = sine (theta r)

sine-1 (0.4402) = sine-1 ( sine (theta r))

26.1 degrees = theta r

The value of 26.1 degrees corresponds to the angle of refraction at boundary #1. Since
boundary #1 is parallel to boundary #2, the angle of refraction at boundary #1 will be the
same as the angle of incidence at boundary #2 (see diagram above). So now repeat the
process in order to solve for the angle of refraction at boundary #2.

Boundary #2:

1.52 * sine (26.1 degrees) = 1.00 * sine (theta r)

1.52 * (0.4402) = 1.00 * sine (theta r)

0.6691 = sine (theta r)

sine-1 (0.6691) = sine-1 ( sine (theta r))

42.0 degrees = theta r


The answers to this problem are 26.1 degrees and 42.0 degrees.

There is an important conceptual idea which is found from an inspection of the above answer.
The ray of light approached the top surface of the layer at 42 degrees and exited through the
bottom surface of the layer with the same angle of 42 degrees. The light ray refracted one
direction upon entering and the other direction upon exiting; the two individual effects have
balanced each other and the ray is moving in the same direction. The important concept is
this:

When light approaches a layer which has the shape of a parallelogram that is bounded on both
sides by the same material, then the angle at which the light enters the material is equal to
the angle at which light exits the layer.
If the layer is not a parallelogram or is not bound on both sides by the same material, then
this will not be the case. Knowing this concept will allow you to conduct a quick check of an
answer in a situation like this.

Example Problem C
A ray of light in air is approaches a triangular piece of crown glass at an angle of 0.00 degrees (as shown in the diagram
at the right). Perform the necessary calculations in order to trace the path of the light ray as it enters and exits the crown
glass. Refer to the table of indices of refraction if necessary.

Solution to Problem C
This problem is even more complicated than Practice Problem B. Like Practice Problem B, there
are two boundaries; but unlike Problem B, the two boundaries are not parallel to each other.
The problem can be treated like a layer problem in which the light refracts upon entering the
glass (boundary #1: air to crown glass) and upon leaving the glass (boundary #2: crown glass
to air).

Despite the complication of there being nonparallel boundaries, the solution begins like the
above problem: a diagram is constructed to assist in the visualization of the physical situation,
the known values are listed, and the unknown value (desired quantity) is identified. This is
shown below:

Diagram: Given: Find:


Boundary #1 Trace path of light.
ni = 1.00 (from table) That is, find at

nr = 1.52 (from table) boundary #1

= 0.0 degrees and

and at

Boundary #2 boundary #2

ni = 1.52 (from table)

nr = 1.00 (from table)

Now list the relevant equation (Snell's Law), substitute known values into the equation, and
perform the proper algebraic steps to solve for the unknown. Begin the process at boundary
#1 and then repeat for boundary #2 until the final answer is found.

Boundary #1:

1.00 * sine (0.0 degrees) = 1.52 * sine (theta r)

0.000 = 1.52 * sine (theta r)

0.000 = sine (theta r)

sine-1 (0.000) = sine-1 ( sine (theta r))

0.00 degrees = theta r

This problem is made easier if you draw upon your conceptual knowledge of what occurs when
a light ray approaches at an angle of incidence of 0-degrees (recall the If I Were an Archer
Fish page). When approaching along the normal, the light ray passes across the boundary
without refracting. If you did not know this, then you would merely recognize it upon
performing your first calculation of the angle of refraction at the first boundary. The fact that
the answer is 0 degrees - the same as the incident angle - means that light did not refract at
this boundary.
The next step demands that the light ray be traced through the triangular block until it
reaches the second boundary. Draw the refracted ray at 0 degrees (i.e., trace the incident ray
straight through the first boundary). At the second boundary, the normal line must be drawn
(labeled N) and the angle of incidence (between the incident ray and the normal) must be
measured. This is shown on the diagram at the right. The measured value of the angle of
incidence at the second boundary is 30.0 degrees. This angle measurement now provides
knowledge of three of the four variables in the Snell's Law equation and allows for the
determination of the fourth variable (the angle of refraction) at the second boundary.

(Note: the given angle measures for the 30-60-90 degree triangle can be used along with the
fact that any three angles of a triangle add to 180 degrees in order to geometrically determine
this angle measure.)

Boundary #2:

1.52 * sine (30.0 degrees) = 1.00 * sine (theta r)

1.52 * (0.5000) = 1.00 * sine (theta r)

0.7600 = sine (theta r)

sine-1 (0.7600) = sine-1 ( sine (theta r))

49.5 degrees = theta r

The refracted ray at the second boundary will exit at an angle of 49.5 degrees from the
normal. This can be measured on the diagram and drawn with a straight edge as shown in the
diagram at the right.

The above three practice problems demonstrate a sampling of the variety of problems which
could be encountered. In thenext part of Lesson 2, we will see one more type of problem.

Check Your Understanding


1. Determine the angle of refraction for the following two refraction problems.

2. Perform the necessary calculations at each boundary in order to trace the path of the light
ray through the following series of layers. Use a protractor and a ruler and show all your work.
3. A ray of light in crown glass exits into air at an angle of 25.0 degrees. Determine the angle
at which the light approached the glass-air boundary. Refer to the table of indices of
refraction if necessary.

4. A ray of light is traveling through air (n = 1.00) towards a lucite block (n = 1.40) in the
shape of a 30-60-90 triangle. Trace the path of the light ray through the lucite block shown in
the diagram below.

Next Section: Determination of n Values


Jump To Lesson 3: Total Internal Reflection
The Mathematics of Refraction
Determination of n Values
In a previous part of Lesson 2, we learned about a mathematical equation relating the two
angles (angles of incidence and refraction) and the indices of refraction of the two materials on
each side of the boundary. The equation is known as the Snell's Law equation and is expressed
as follows.
where ("theta i") = angle of incidence

("theta r") = angle of refraction

ni = index of refraction of the incident medium

nr = index of refraction of the refractive medium

As with any equation in physics, the Snell's Law equation is valued for its predictive ability. If
any three of the four variables in the equation are known, the fourth variable can be predicted
if appropriate problem-solving skills are employed. The task of using the equation to solve
several varieties of problems was thoroughly discussed in a previous part of this Lesson 2. In a
similar manner, the equation can be used to determine the index of refraction of a material if
the path of light through the material is known. In this part of Lesson 2, we will investigate the
details of this task.

A common lab performed in a Physics class involves the determination of the index of
refraction of an unknown material. Typically, a series of transparent objects (glass and lucite
squares, rectangles and triangles) made of an unknown material are distributed and the task
is assigned to determine the index of refraction of each unknown material. Of course, the task
of determining the index of refraction value of an unknown material would be easy if the other
three quantities in the Snell's Law equation are known. Thus, a line of sight method or a laser
beam is used to determine the path of the light into the material, through the material and out
of the material to the students' eyes. The diagram depicts a typical path of one such ray
through a square block. Observe that the light encounters two boundaries - the boundary
upon entering the glass and the boundary upon exiting the glass. The ray of light refracts at
each boundary. By measuring the angles of incidence and refraction and using the index of
refraction of air, the index of refraction of the unknown material can be found. Calculations can
be performed at each boundary and the results can be averaged. The data table below
represent sample data for the Index of Refraction Lab; the listed values correspond to the
diagram at the right.

Sample Data
sine sine Calculated n
value

Light Ray 45 deg. 24 deg. 0.7071 0.4067 1.74


Entering

Material

Light Ray 24 deg. 45 deg. 0.4067 0.7071 1.74


Exiting

Material

1.74
Ave. n

Value --->

The accompanying work for this lab are as follows:

First Boundary: Light Ray Entering Material


Diagram: Known: Find:
nr = ???
ni = 1.00 (from table)

= 45 degrees

= 24 degrees

1.00 * sine(45 deg.) = nr * sine(24 deg.)

0.7071 = 0.4067 * nr

(0.7071)/(0.4067) = nr

1.74 = nr

Second Boundary: Light Ray Exiting Material

Diagram: Known: Find:


ni = ???
nr = 1.00 (from table)
= 45 degrees

= 24 degrees

ni * sine(24 deg.) = 1.000 * sine(45 deg.)

ni * 0.4067 = 0.7071

ni = (0.7071)/(0.4067)

ni = 1.74

Observe in the data table above the two calculated n values are the same. This should not be
a surprise since the unknown material only has one index of refraction value. If different
values were obtained, it would be the result of some form of measurement error in the
retrieval of data.

This method of determining the index of refraction of an unknown material is one more
application of the Snell's Law equation. Test your ability to use the equation in this manner by
answering the following questions.

Check Your Understanding

1. Cal Culator is performing an experiment to determine the index of refraction of an unknown


material (in the shape of a 45-45-90 triangle). Cal determines that the light follows the path
as shown on the diagram below. Use this path, a protractor, a calculator and Snell's Law to
determine the index of refraction of the unknown material.
2. The path of a light ray through an unknown material is shown in the diagram below. Make
some measurements and determine the index of refraction of the material.

3. Light traveling through air (n = 1.00) is incident upon a triangular block made of an
unknown material. The path of the light through the material is shown in the diagram below.
Using a protractor and a calculator, determine the index of refraction of the unknown material.
4. Light traveling through air (n = 1.00) is incident upon a 60-60-60 triangular block (the
triangle is equilateral; the sides make 60-degree angles with each other) made of an unknown
material. The path of the light through the material is shown in the diagram below. Using a
protractor and a calculator, determine the index of refraction of the unknown material.

Total Internal Reflection


Boundary Behavior Revisited
Earlier in this unit, the boundary behavior of light waves was discussed. It was
mentioned that a light wave doesn't just stop when it reaches the end of the
medium. Rather, the light wave undergoes certain behaviors when it encounters
the end of the medium - such behaviors include reflection,
transmission/refraction, and diffraction. InUnit 13 of The Physics Classroom Tutorial, the
primary focus was the reflective behavior of light waves at the boundary. In this unit, our
primary interest has been the refractive behavior of light waves at the boundary. In Lesson 3,
we will investigate the connection between light reflection and light refraction.

A light wave, like any wave, is an energy-transport phenomenon. A light wave transports
energy from one location to another. When a light wave strikes a boundary between two
distinct media, a portion of the energy will be transmitted into the new medium and a portion
of the energy will be reflected off the boundary and stay within the original medium. The
actual percentage of energy which is transmitted and reflected is dependent upon a number of
variables; these will be discussed as we proceed through Lesson 3. For now, our concern is to
review and internalize the basic concepts and terminology associated with boundary behavior.
Reflection of a light wave involves the bouncing of a light wave off the boundary, while
refraction of a light wave involves the bending of the path of a light wave upon crossing a
boundary and entering a new medium. Both reflection and refraction involve a change in
direction of a wave, but only refraction involves a change in medium.

The diagram at the right shows several wavefronts approaching a boundary between two
media. These wavefronts are referred to as the incident waves and the ray which points in the
direction which they are traveling is referred to as the incident ray. The incident ray is drawn
in blue on the diagram at the right. Notice on the diagram that the incident ray leads into two
other rays at the point of incidence with the boundary. The reflected waves are the waves
which bounce off the boundary and head back upwards and the reflected ray is the ray which
points in the direction which the reflected waves are traveling. The reflected ray is drawn in
green on the diagram at the right. The refracted waves are the waves which are transmitted
across the boundary and continues moving downwards, only at a different angle than before.
The refracted ray is the ray which points in the direction which the refracted waves are
traveling. The refracted ray is drawn in red on the diagram at the right. At the point of
incidence (the point where the incident ray strikes the boundary), a normal line is drawn.
The normal line is always drawn perpendicular to the surface at the point of incidence. The
normal line creates a variety of angles with the light rays; these angles are important and are
given special names. The angle between the incident ray and the normal is the angle of
incidence. The angle between the reflected ray and the normal is the angle of reflection.
And the angle between the refracted ray and the normal is the angle of refraction.
The fundamental law which governs the reflection of light is called the law of reflection.
Whether the light is reflecting off a rough surface or a smooth surface, a curved surface or a
planar surface, the light ray follows the law of reflection. The law of reflection states that

When a light ray reflects off a surface, the angle of incidence is equal to the angle of
reflection.
The fundamental law which governs the refraction of light is Snell's Law. Snell's Lawstates
that

When a light ray is transmitted into a new medium, the relationship between the angle of
incidence and the angle of refraction is given by the following equation

where the ni and nr values represent the indices of refraction of the incident and the refractive
medium respectively.

As we proceed through this Lesson, we will see that there is a connection between the
reflection and the refraction of light. Each of these two behaviors usually occur together. But
as we will see, there are two conditions, which when both met, will cause the light waves to
undergo reflection without any accompanying refraction.

Total Internal Reflection


Total Internal Reflection
A common Physics lab is to sight through the long side of an isosceles triangle at a pin or
other object held behind the opposite face. When done so, an unusual observation - a
discrepant event - is observed. The diagram on the left below depicts the physical situation. A
ray of light entered the face of the triangular block at a right angle to the boundary. This ray of
light passes across the boundary without refraction since it was incident along the normal
(recall the If I Were An Archer Fish page). The ray of light then travels in a straight line
through the glass until it reaches the second boundary. Now instead of transmitting across this
boundary, all of the light seems to reflect off the boundary and transmit out the opposite face
of the isosceles triangle. This discrepant event bothers many as they spend several minutes
looking for the light to refract through the second boundary. Then finally, to their amazement,
they looked through the third face of the block and clearly see the ray. What happened? Why
did light not refract through the second face?
The phenomenon observed in this part of the lab is known as total internal reflection.Total
internal reflection, or TIR as it is intimately called, is the reflection of the total amount of
incident light at the boundary between two medium. TIR is the topic of focus in Lesson 3.

To understand total internal reflection, we will begin with a thought experiment. Suppose that
a laser beam is submerged in a tank of water (don't do this at home) and pointed upwards
towards water-air boundary. Then suppose that the angle at which the beam is directed
upwards is slowly altered, beginning with small angles of incidence and proceeding towards
larger and larger angles of incidence. What would be observed in such an experiment? If we
understand the principles of boundary behavior, we would expect that we would observe both
reflection and refraction. And indeed, that is what is observed (mostly). But that's not the only
observation which we could make. We would also observe that the intensity of the reflected
and refracted rays do not remain constant. At angle of incidence close to 0 degrees, most of
the light energy is transmitted across the boundary and very little of it is reflected. As the
angle is increased to greater and greater angles, we would begin to observe less refraction
and more reflection. That is, as the angle of incidence is increased, the brightness of the
refracted ray decreases and the brightness of the reflected ray increases. Finally, we would
observe that the angles of the reflection and refraction are not equal. Since the light waves
would refract away from the normal (a case of the SFA principle of refraction), the angle of
refraction would be greater than the angle of incidence. And if this is the case, the angle of
refraction would also be greater than the angle of reflection (since the angles of reflection and
incidence are the same). As the angle of incidence is increased, the angle of refraction would
eventually reach a 90-degree angle. These principles are depicted in the diagram below.
The maximum possible angle of refraction is 90-degrees. If you think about it (a practice
which always helps), you recognize that if the angle of refraction were greater than 90
degrees, then the refracted ray would lie on the incident side of the medium - that's just not
possible. So in the case of the laser beam in the water, there is some specific value for the
angle of incidence (we'll call it the critical angle) which yields an angle of refraction of 90-
degrees. This particular value for the angle of incidence could be calculated usingSnell's
Law (ni = 1.33, nr = 1.000, = 90 degrees, = ???) and would be found to be 48.6 degrees.
Any angle of incidence which is greater than 48.6 degrees would not result in refraction.
Instead, when the angles of incidence is greater than 48.6 degrees (the critical angle), all of
the energy (the total energy) carried by the incident wave to the boundary stays within the
water (internal to the original medium) and undergoesreflection off the boundary. When this
happens, total internal reflection occurs.

Two Requirements for Total Internal Reflection

Total internal reflection (TIR) is the phenomenon which involves the reflection of all the
incident light off the boundary. TIR only takes place when both of the following two conditions
are met:

• the light is in the more dense medium and approaching the less dense medium.
• the angle of incidence is greater than the so-called critical angle.

Total internal reflection will not take place unless the incident light is traveling within the
more optically dense medium towards the less optically dense medium. TIR will happen for
light traveling from water towards air, but it will not happen for light traveling from air towards
water. TIR would happen for light traveling from water towards air, but it will not happen for
light traveling from water (n=1.333) towards crown glass (n=1.52). TIR occurs because the
angle of refraction reaches a 90-degree angle before the angle of incidence reaches a 90-
degree angle. The only way for the angle of refraction to be greater than the angle of
incidence, is for light to bend away from the normal. Since light only bends away from the
normal when passing from a more dense medium into a less dense medium, then this would
be a necessary condition for total internal reflection.

Total internal reflection only occurs with large angles of incidence. Question: How large is
large? Answer: larger than the critical angle. As mentioned above, the critical angle for the
water-air boundary is 48.6 degrees. So for angles of incidence greater than 48.6-degrees, TIR
occurs. But 48.6 degrees is the critical angle only for the water-air boundary. The actual value
of the critical angle is dependent upon the two materials on either side of the boundary. For
the crown glass-air boundary, the critical angle is 41.1 degrees. For the diamond-air boundary,
the critical angle is 24.4 degrees. For the diamond-water boundary, the critical angle is 33.4
degrees. The critical angle is different for different media. In the next part of Lesson 3, we will
investigate how to determine the critical angle for any two materials. For now, let's internalize
the idea that TIR can only occur if the angle of incidence is greater than the critical angle for
the particular combination of materials.

Light Piping and Optical Fibers

Total internal reflection is often demonstrated in a Physics class through a variety of


demonstrations. In one such demonstration, a beam of laser light is directed into a coiled
plastic thing-a-ma jig. The plastic served as a light pipe, directing the light through the coils
until it finally exits out the opposite end. Once the light entered the plastic, it was in the more
dense medium. Every time the light approached the plastic-air boundary, it is approaching at
angles greater than the critical angle. The two conditions necessary for TIR are met, and all of
the incident light at the plastic-air boundary stays internal to the plastic and undergoes
reflection. And with the room lights off, every student becomes quickly aware of the ancient
truth that Physics is better than drugs.
This demonstration helps to illustrate the principle by which optical fibers work.
The use of a long strand of plastic (or other material such as glass) to pipe light
from one end of the medium to the other is the basis for modern day use
of optical fibers. Optical fibers are are used in communication systems and
micro-surgeries. Since total internal reflection takes place within the fibers, no
incident energy is ever lost due to the transmission of light across the boundary.
The intensity of the signal remains constant.

Another common Physics demonstration involves the use of a large jug filled with water and a
laser beam. The jug has a pea-sized hole drilled in its side such that when the cork is removed
from the top of the jug, water begins to stream out the jug's side. The beam of laser light is
then directed into the jug from the opposite side of the hole, through the water and into the
falling stream. The laser light exits the jug through the hole but is still in the water. As the
stream of water begins to fall as a projectile along a parabolic path to the ground, the laser
light becomes trapped within the water due to total internal reflection. Being in the more
dense medium (water) and heading towards a boundary with a less dense medium (air), and
being at angles of incidence greater than the critical angle, the light never leaves the stream
of water. In fact, the stream of water acts as a light pipe to pipe the laser beam along its
trajectory. Once more, students viewing the demonstration are convinced of the fact that
Physics is better than drugs.

Check Your Understanding

1. For each combination of media, which light ray (A or B) will undergo total internal reflection
if the incident angle is gradually increased?
Total Internal Reflection
The Critical Angle
In the previous part of Lesson 3, the phenomenon of total internal reflection was introduced.
Total internal reflection (TIR) is the phenomenon which involves the reflection of all the
incident light off the boundary. TIR only takes place when both of the following two conditions
are met:

• a light ray is in the more dense medium and approaching the less dense
medium.
• the angle of incidence for the light ray is greater than the so-called critical
angle.

In our introduction to TIR, we used the example of light traveling through water towards the
boundary with a less dense material such as air. When the angle of incidence in water reaches
a certain critical value, the refracted ray lies along the boundary, having an angle of refraction
of 90-degrees. This angle of incidence is known as the critical angle; it is the largest angle of
incidence for which refraction can still occur. For any angle of incidence greater than the
critical angle, light will undergo total internal reflection.

So the critical angle is defined as the angle of incidence which provides an angle of refraction
of 90-degrees. Make particular note that the critical angle is an angle of incidence value. For
the water-air boundary, the critical angle is 48.6-degrees. For the crown glass-water boundary,
the critical angle is 61.0-degrees. The actual value of the critical angle is dependent upon the
combination of materials present on each side of the boundary.

Let's consider two different media - creatively named medium i (incident medium) and
medium r (refractive medium). The critical angle is the which gives a value of 90-
degrees. If this information is substituted into Snell's Law equation, a generic equation for
predicting the critical angle can be derived. The derivation is shown below.

ni *• sine( ) = nr • sine ( )
ni • sine( ) = nr • sine (90 degrees)
ni • sine( ) = nr

sine( ) = nr/ni

= sine-1 (nr/ni) = invsine (nr/ni)

The critical angle can be calculated by taking the inverse-sine of the ratio of the indices of
refraction. The ratio of nr/ni is a value less than 1.0. In fact, for the equation to even give a
correct answer, the ratio of nr/ni must be less than 1.0. Since TIR only occurs if the refractive
medium is less dense than the incident medium, the value of ni must be greater than the value
of nr. If at any time the values for the numerator and denominator become accidentally
switched, the critical angle value cannot be calculated. Mathematically, this would involve
finding the inverse-sine of a number greater than 1.00 - which is not possible. Physically, this
would involve finding the critical angle for a situation in which the light is traveling from the
less dense medium into the more dense medium - which again, is not possible.

This equation for the critical angle can be used to predict the critical angle for any boundary,
provided that the indices of refraction of the two materials on each side of the boundary are
known. Examples of its use are shown below:

Example A
Calculate the critical angle for the crown glass-air boundary. Refer to the table of indices of refraction if necessary.

The solution to the problem involves the use of the above equation for the critical angle.

= sin-1 (nr/ni) = invsine (nr/ni)


= sin-1 (1.000/1.52) = 41.1 degrees

Example B
Calculate the critical angle for the diamond-air boundary. Refer to the table of indices of refraction if necessary.

The solution to the problem involves the use of the above equation for the critical angle.

= sin-1 (nr/ni) = invsine (nr/ni)


= sin-1 (1.000/2.42) = 24.4 degrees

TIR and the Sparkle of Diamonds


Relatively speaking, the critical angle for the diamond-air boundary is an extremely small
number. Of all the possible combinations of materials which could interface to form a
boundary, the combination of diamond and air provides one of the largest difference in the
index of refraction values. This means that there will be a very small nr/ni ratio and
subsequently a small critical angle. This peculiarity about the diamond-air boundary plays an
important role in the brilliance of a diamond gemstone. Having a small critical angle, light has
the tendency to become "trapped" inside of a diamond once it enters. A light ray will typically
undergo TIR several times before finally refracting out of the diamond. Because the diamond-
air boundary has such a small critical angle (due to diamond's large index of refraction), most
rays approach the diamond at angles of incidence greater than the critical angle. This gives
diamond a tendency to sparkle. The effect can be enhanced by the cutting of a diamond
gemstone with a strategically planned shape. The diagram below depicts the total internal
reflection within a diamond gemstone with a strategic and a non-strategic cut.

Check Your Understanding

1. Suppose that the angle of incidence of a laser beam in water and heading towards air is
adjusted to 50-degrees. Use Snell's law to calculate the angle of refraction? Explain your result
(or lack of result).
2. Aaron Agin is trying to determine the critical angle of the diamond-glass
surface. He looks up the index of refraction values of diamond (2.42) and crown
glass (1.52) and then tries to compute the critical angle by taking the

invsine(2.42/1.52).
Unfortunately, Aaron's calculator keeps telling him he has an ERROR! Aaron hits
the calculator and throws it own the ground a few times; he then repeats the calculation with
the same result. He then utters something strange about the pizza he had slopped on it the
evening before and runs out of the library with a disappointed disposition. What is Aaron's
problem? (That is, what is the problem with his method of calculating the critical angle?)

3. Calculate the critical angle for an ethanol-air boundary. Refer to the table of indices of
refraction if necessary.

4. Calculate the critical angle for an flint glass(light)-air boundary. Refer to the table of indices
of refraction if necessary.

5. Calculate the critical angle for a diamond-crown glass boundary. Refer to the table of indices
of refraction if necessary.

6. Some optical instruments, such as periscopes and binoculars use trigonal prisms instead of
mirrors to reflect light around corners. Light typically enters perpendicular to the face of the
prism, undergoes TIR off the opposite face and then exits out the third face. Why do you
suppose the manufacturer prefers the use of prisms instead of mirrors?
Interesting Refraction Phenomena
Dispersion of Light by Prisms
In the Light and Color unit of The Physics Classroom Tutorial, the visible light spectrum was
introduced and discussed. Visible light, also known as white light, consists of a collection of
component colors. These colors are often observed as light passes through a triangular prism.
Upon passage through the prism, the white light is separated into its component colors - red,
orange, yellow, green, blue and violet. The separation of visible light into its different colors is
known as dispersion. It was mentioned in the Light and Color unit that each color is
characteristic of a distinct wave frequency; and different frequencies of light waves will bend
varying amounts upon passage through a prism. In this unit, we will investigate the dispersion
of light in more detail, pondering the reasons why different frequencies of light bend or refract
different amounts when passing through the prism.

Earlier in this unit, the concept of optical density was introduced. Different materials are
distinguished from each other by their different optical densities. The optical density is simply
a measure of the tendency of a material to slow down light as it travels through it. As
mentioned earlier, a light wave traveling through a transparent material interacts with the
atoms of that material. When a light wave impinges upon an atom of the material, it is
absorbed by that atom. The absorbed energy causes the electrons in the atom to vibrate. If
the frequency of the light wave does not match the resonance frequency of the vibrating
electrons, then the light will be reemitted by the atom at the same frequency at which it
impinged upon it. The light wave then travels through the interatomic vacuum towards the
next atom of the material. Once it impinges upon the next atom, the process of absorption
and reemission is repeated.

The optical density of a material is the result of the tendency of the atoms of a material to
maintain the absorbed energy of the light wave in the form of vibrating electrons before
reemitting it as a new electromagnetic disturbance. Thus, while a light wave travels through a
vacuum at a speed of c (3.00 x 108 m/s), it travels through a transparent material at speeds
less than c. The index of refraction value (n) provides a quantitative expression of the optical
density of a given medium. Materials with higher index of refraction values have a tendency
to hold onto the absorbed light energy for greater lengths of time before reemitting it to the
interatomic void. The more closely that the frequency of the light wave matches the resonant
frequency of the electrons of the atoms of a material, the greater the optical density and the
greater the index of refraction. A light wave would be slowed down to a greater extent when
passing through such a material

What was not mentioned earlier in this unit is that the index of refraction values are
dependent upon the frequency of light. For visible light, the n value does not show a large
variation with frequency, but nonetheless it shows a variation. For instance, the nvalue for
frequencies of violet light is 1.53; and the n value for frequencies of red light is 1.51. The
absorption and reemission process causes the higher frequency (lower wavelength) violet light
to travel slower through crown glass than the lower frequency (higher wavelength) red light. It
is this difference in n value for the varying frequencies(and wavelengths) which causes the
dispersion of light by a triangular prism. Violet light, being slowed down to a greater extent by
the absorption and reemission process, refracts more than red light. Upon entry of white light
at the first boundary of a triangular prism, there will be a slight separation of the white light
into the component colors of the spectrum. Upon exiting the triangular prism at the second
boundary, the separation becomes even greater andROYGBIV is observed in all its splendor.

The Angle of Deviation

The amount of overall refraction caused by the passage of a light ray through a prism is often
expressed in terms of the angle of deviation ( ). The angle of deviation is the angle made
between the incident ray of light entering the first face of the prism and the refracted ray
which emerges from the second face of the prism. Because of the different indices of refraction
for the different wavelengths of visible light, the angle of deviation varies with wavelength.
Colors of the visible light spectrum which have shorter wavelengths (BIV) will deviated more
from their original path than the colors with longer wavelengths (ROY). The emergence of
different colors of light from a triangular prism at different angles leads an observer to see the
component colors of visible light separated from each other.
Of course the discussion of the dispersion of light by triangular prisms begs the following
question: Why doesn't a square or rectangular prism cause the dispersion of a narrow beam of
white light? The short answer is that it does. The long answer is provided in the following
discussion and illustrated by the diagram below.

Suppose that a flashlight could be covered with black paper with a slit across it so as to create
a beam of white light. And suppose that the beam of white light with its component colors
unseparated were directed at an angle towards the surface of a rectangular glass prism. As
would be expected, the light would refract towards the normal upon entering the glass and
away from the normal upon exiting the glass. But since the violet light has a shorter
wavelength, it would refract more than the longer wavelength red light. The refraction of light
at the entry location into the rectangular glass prism would cause a little separation of the
white light. However, upon exiting the glass prism, the refraction takes place in the opposite
direction. The light refracts away from the normal, with the violet light bending a bit more
than the red light. Unlike the passage through the triangular prism with non-parallel sides,
there is no overall angle of deviation for the various colors of white light. Both the red and the
violet components of light are traveling in the same direction as they were traveling before
entry into the prism. There is however a thin red fringe present on one end of the beam and
thin violet fringe present on the opposite side of the beam. This fringe is evidence of
dispersion. Because there is a different angle of deviation of the various components of white
light after transmission across the first boundary, the violet is separated ever so slightly from
the red. Upon transmission across the second boundary, the direction of refraction is reversed;
yet because the violet light has traveled further downward when passing through the rectangle
it is the primary color present in the lower edge of the beam. The same can be said for red
light on the upper edge of the beam.

Dispersion of light provides evidence for the existence of a spectrum of wavelengths present in
visible light. It is also the basis for understanding the formation of rainbows.Rainbow
formation is the next topic of discussion in Lesson 4.
Interesting Refraction Phenomena
Rainbow Formation
One of nature's most splendid masterpieces is the rainbow. A rainbow is
an excellent demonstration of the dispersion of light and one more piece
of evidence that visible light is composed of a spectrum of wavelengths,
each associated with a distinct color. To view a rainbow, your back must be to the sun as you
look at an approximately 40 degree angle above the ground into a region of the atmosphere
with suspended droplets of water or even a light mist. Each individual droplet of water acts as
a tiny prism which both disperses the light and reflects it back to your eye. As you sight into
the sky, wavelengths of light associated with a specific color arrive at your eye from the
collection of droplets. The net effect of the vast array of droplets is that a circular arc
of ROYGBIV is seen across the sky. But just exactly how do the droplets of water disperse and
reflect the light? And why does the pattern always appear as ROYGBIV from top to bottom?
These are the questions which we will seek to understand on this page of The Physics
Classroom Tutorial. To understand these questions, we will need to draw upon our
understanding of refraction, internal reflection and dispersion.

The Path of Light Through a Droplet

A collection of suspended water droplets in the atmosphere serve as a refractor of light. The
water represents a medium with a different optical density than the surrounding air. Light
waves refract when they cross over the boundary from one medium to another. The decrease
in speed upon entry of light into a water droplet causes a bending of the path of light towards
the normal. And upon exiting the droplet, light speeds up and bends away from the normal.
The droplet causes a deviation in the path of light as it enters and exits the drop.

There are countless paths by which light rays from the sun can pass through a drop. Each path
is characterized by this bending towards and away from the normal. One path of great
significance in the discussion of rainbows is the path in which light refracts into the droplet,
internally reflects, and then refracts out of the droplet. The diagram at the right depicts such a
path. A light ray from the sun enters the droplet with a slight downward trajectory. Upon
refracting twice and reflecting once, the light ray is dispersed and bent downward towards an
observer on earth's surface. Other entry locations into the droplet may result in similar paths
or even in light continuing through the droplet and out the opposite side without significant
internal reflection. But for the entry location shown in the diagram at the right, there is an
optimal concentration of light exiting the airborne droplet at an angle towards the ground. As
in the case of the refraction of light through prisms with nonparallel sides, the refraction of
light at two boundaries of the droplet results in the dispersion of light into a spectrum of
colors. The shorter wavelength blue and violet light refract a slightly
greater amount than the longer wavelength red light. Since the
boundaries are not parallel to each other, the double refraction results in
a distinct separation of the sunlight into its component colors.

The angle of deviation between the incoming light rays from the sun and the refracted rays
directed to the observer's eyes is approximately 42 degrees for the red light. Because of the
tendency of shorter wavelength blue light to refract more than red light, its angle of deviation
from the original sun rays is approximately 40 degrees. As shown in the diagram, the red light
refracts out of the droplet at a steeper angle toward an observer on the ground. There are a
multitude of paths by which the original ray can pass through a droplet and and subsequently
angle towards the ground. Some of the paths are dependent upon which part of the droplet
the incident rays contact. Other paths are dependent upon the location of the sun in the sky
and the subsequent trajectory of the incoming rays towards the droplet. Yet the greatest
concentration of outgoing rays is found at these 40-42 degree angles of deviation. At these
angles, the dispersed light is bright enough to result in a rainbow display in the sky. Now that
we understand the path of light through an individual droplet, we can approach the topic of
how the rainbow forms.

The Formation of the Rainbow

A rainbow is most often viewed as a circular arc in the sky. An observer on the ground
observes a half-circle of color with red being the color perceived on the outside or top of the
bow. Those who are fortunate enough to have seen a rainbow from an airplane in the sky may
know that a rainbow can actually be a complete circle. Observers on the ground only view the
top half of the circle since the bottom half of the circular arc is prevented by the presence of
the ground (and the rather obvious fact that suspended water droplets aren't present below
ground). Yet observers in an airborne plane can often look both upward and downward to view
the complete circular bow.

The circle (or half-circle) results because there are a collection of suspended droplets in the
atmosphere which are capable concentrating the dispersed light at angles of deviation of 40-
42 degrees relative to the original path of light from the sun. These droplets actually form a
circular arc, with each droplet within the arc dispersing light and reflecting it back towards the
observer. Every droplet within the arc is refracting and dispersing the entire visible light
spectrum (ROYGBIV). As described above, the red light is refracted out of a droplet at steeper
angles towards the ground than the blue light. Thus, when an observer sights at a steeper
angle with respect to the ground, droplets of water within this line of sight are refracting the
red light to the observer's eye. The blue light from these same droplets is directed at a less
steep angle and is directed along a trajectory which passes over the observer's head. Thus,
it is the red light which is seen when looking at the steeper angles relative to the ground.
Similarly, when sighting at less steep angles, droplets of water within this line of sight are
directing blue light to the observer's eye while the red light is directed downwards at a more
steep angle towards the observer's feet. This discussion explains why it is the red light which
is observed at the top and on the outer perimeter of a rainbow and the blue light which is
observed on the bottom and the inner perimeter of the rainbow.

Rainbows are not limited to the dispersion of light by raindrops. The splashing of water at the
base of a waterfall caused a mist of water in the air which often results in the formation of
rainbows. A backyard water sprinkler is another common source of a rainbow. Bright sunlight,
suspended droplets of water and the proper angle of sighting are the three necessary
components for viewing one of nature's most splendid masterpieces.

Interesting Refraction Phenomena


Mirages
Most of our discussion of refraction in this unit has pertained to the refraction of light at a
distinct boundary. As light is transmitted across the boundary from one material to another,
there is a change in speed which causes a change in direction of the light wave. The
boundaries which we have been focusing on have been distinct interfaces between two
recognizably different materials. The boundary between the glass of a fish tank and the
surrounding air or the boundary between the water in a pool and the surrounding air are
examples of distinct interfaces between two recognizably different materials.
It has been mentioned in our discussion that the refraction or bending of light occurs at the
boundary between two materials; and once a light wave has crossed the boundary it travels in
a straight line. The discussion has presumed that the medium is a uniformmedium. A uniform
medium is a medium whose optical density is everywhere the same within the medium. A
uniform medium is the same everywhere from its top boundary to its bottom boundary and
from its left boundary to its right boundary. But not every medium is a uniform medium, and
the fact that air can sometimes form a nonuniform medium leads to an interesting refraction
phenomenon - the formation of mirages.

A mirage is an optical phenomenon which creates the illusion of water and results from the
refraction of light through a nonuniform medium. Mirages are most commonly observed on
sunny days when driving down a roadway. As you drive down the roadway, there appears to
be a puddle of water on the road several yards (maybe one-hundred yards) in front of the car.
Of course, when you arrive at the perceived location of the puddle, you recognize that the
puddle is not there. Instead, the puddle of water appears to be another one-hundred yards in
front of you. You could carefully match the perceived location of the water to a roadside
object; but when you arrive at that object, the puddle of water is still not on the roadway. The
appearance of the water is simply an illusion.

Mirages occur on sunny days. The role of the sun is to heat the roadway to high temperatures.
This heated roadway in turn heats the surrounding air, keeping the air just above the roadway
at higher temperatures than that day's average air temperature. Hot air tends to be less
optically dense than cooler air. As such, a nonuniform medium has been created by the
heating of the roadway and the air just above it. While light will travel in a straight line
through a uniform medium, it will refract when traveling through a nonuniform medium. If a
driver looks down at the roadway at a very low angle (that is, at a position nearly one-
hundred yards away), light from objects above the roadway will follow a curved path to the
driver's eye as shown in the diagram below.

Light which is traveling downward into this less optically dense air begins to speed up. Though
there isn't a distinct boundary between two media, there is a change in speed of a light wave.
As expected, a change in speed is accompanied by a change in direction. If there were a
distinct boundary between two media, then there would be a bending of this light ray away
from the normal. For this light ray to bend away from the normal(towards the boundary), the
ray would begin to bend more parallel to the roadway and then bend upwards towards the
cooler air. As such, a person in a car sighting downward at the roadway will see an object
located above the roadway.

Of course, this is not a usual event. When was the last time that you looked downward at a
surface and saw an object above the surface? While not a usual event, it does happen. For
instance, suppose you place a mirror on the floor and look downward at the floor; you will see
objects located above the floor due to the reflection of light by the mirror. Even a glass window
placed on the floor will reflect light from objects above the floor. If you look downward at the
glass window at a low enough angle, then you will see objects located above the floor. Or
suppose that you are standing on the shore of a calm pond and look downward at the water;
you might see objects above the pond due to the reflection of light by the water.

So when you experience this sunny day phenomenon, your mind must quickly make sense of
how you can look downward at the roadway and see an object located above the road. In the
process of making sense of this event, your mind draws upon past experiences. Searching the
database of stored experiences, your mind is interested in an explanation of why the eye can
sight downward at a surface and see an object which is located above the surface. In the
process of searching, it comes up with three possible explanations based upon past
experiences. Your mind subtly ponders these three options.

• There is a mirror on the road. Someone must have for some reason placed a
mirror on the road. The mirror is reflecting light and that is why I see an
image of the oncoming truck when I look downward at the road.
• There is a glass window on the road. My gosh, do you believe it! Someone
has left a glass window on the road. The glass window is reflecting light and
that is why I see an image of the oncoming truck when I look downward at
the road.
• There is water on the road. It must have rained last night and there is a
puddle of water left on the road. The water is reflecting light and that is why I
see an image of the oncoming truck when I look downward at the road.

Of the three possible explanations of the image of the truck, only one makes a lot of sense to
the mind - there is water on the road. After all, while both glass windows and mirrors can
reflect light, nowhere in your mind's database of past experiences is there an account of a
mirror or glass window being seen on a roadway. Yet there are plenty of times that a water
puddle has been observed to be present on a roadway. Smart person that you are, you then
conclude that there is a puddle of water on the road which is causing you to see objects
located above the road when you sight downward at the road. The illusion is complete.
Jump To Lesson 5: Image Formation by Lenses
Image Formation by Lenses
The Anatomy of a Lens
If a piece of glass or other transparent material takes on the appropriate shape, it is possible
that parallel incident rays would either converge to a point or appear to be diverging from a
point. A piece of glass which has such a shape is referred to as a lens.

A lens is merely a carefully ground or molded piece of transparent material which refracts
light rays in such as way as to form an image. Lenses can be thought of as a series of tiny
refracting prisms, each of which refracts light to produce their own image. When these prisms
act together, they produce a bright image focused at a point.

There are a variety of types of lenses. Lenses differ from one another in terms of their shape
and the materials from which they are made. Our focus will be upon lenses which are
symmetrical across their horizontal axis - known as the principal axis. In this unit, we will
categorize lenses as converging lenses and diverging lenses. A converging lensis a lens
which converges rays of light which are traveling parallel to its principal axis. Converging
lenses can be identified by their shape; they are relatively thick across their middle and thin at
their upper and lower edges. A diverging lens is a lens which diverges rays of light which are
traveling parallel to its principal axis. Diverging lenses can also be identified by their shape;
they are relatively thin across their middle and thick at their upper and lower edges.
A double convex lens is symmetrical across both its horizontal and vertical axis. Each of the
lens' two faces can be thought of as originally being part of a sphere. The fact that a double
convex lens is thicker across its middle is an indicator that it will converge rays of light which
travel parallel to its principal axis. A double convex lens is a converging lens. A double
concave lens is also symmetrical across both its horizontal and vertical axis. The two faces of
a double concave lens can be thought of as originally being part of a sphere. The fact that a
double concave lens is thinner across its middle is an indicator that it will diverge rays of light
which travel parallel to its principal axis. A double concave lens is a diverging lens. These two
types of lenses - a double convex and a double concave lens will be the only types of lenses
which will be discussed in this unit of The Physics Classroom Tutorial.

As we begin to discuss the refraction of light rays and the formation of images by these two
types of lenses, we will need to use a variety of terms. Many of these terms should be familiar
to you because they have already been discussed during Unit 13. If you are uncertain of the
meaning of the terms, spend some time reviewing them so that their meaning is firmly
internalized in your mind. They will be essential as we proceed through Lesson 5. These terms
describe the various parts of a lens and include such words as

Principal axis Vertical Plane


Focal Point Focal Length
If a symmetrical lens is thought of as being a slice of a sphere, then there would be a line
passing through the center of the sphere and attaching to the mirror in the exact center of the
lens. This imaginary line is known as the principal axis. A lens also has an
imaginary vertical axis which bisects the symmetrical lens into halves. As mentioned above,
light rays incident towards either face of the lens and traveling parallel to the principal axis will
either converge or diverge. If the light rays converge (as in a converging lens), then they will
converge to a point. This point is known as the focal point of the converging lens. If the light
rays diverge (as in a diverging lens), then the diverging rays can be traced backwards until
they intersect at a point. This intersection point is known as the focal point of a diverging
lens. The focal point is denoted by the letter F on the diagrams below. Note that each lens has
two focal points - one on each side of the lens. Unlike mirrors, lenses can allow light to pass
through either face, depending on where the incident rays are coming from. Subsequently,
every lens has two possible focal points. The distance from the mirror to the focal point is
known as thefocal length (abbreviated by f). Technically, a lens does not have a center of
curvature (at least not one which has any importance to our discussion). However a lens does
have an imaginary point which we refer to as the 2F point. This is the point on the principal
axis which is twice as far from the vertical axis as the focal point is.

As we discuss the characteristics of images produced by converging and diverging lenses,


these vocabulary terms will become increasingly important. Remember that this page is here
and refer to it as often as needed.

Image Formation by Lenses


Refraction by Lenses
We have already learned that a lens is a carefully ground or molded piece of transparent
material which refracts light rays in such as way as to form an image. Lenses serve to refract
light at each boundary. As a ray of light enters a lens, it is refracted; and as the same ray of
light exits the lens, it is refracted again. The net effect of the refraction of light at these two
boundaries is that the light ray has changed directions. Because of the special geometric
shape of a lens, the light rays are refracted such that they form images. Before we
approach the topic of image formation, we will investigate the refractive ability of converging
and diverging lenses.

First lets consider a double convex lens. Suppose that several rays of light approach the lens;
and suppose that these rays of light are traveling parallel to the principal axis. Upon reaching
the front face of the lens, each ray of light will refract towards the normal to the surface. At
this boundary, the light ray is passing from air into a more dense medium (usually plastic or
glass). Since the light ray is passing from a medium in which it travels fast (less optically
dense) into a medium in which it travels relatively slow (moreoptically dense), it will bend
towards the normal line. This is the FST principle of refraction. This is shown for two incident
rays on the diagram below. Once the light ray refracts across the boundary and enters the
lens, it travels in a straight line until it reaches the back face of the lens. At this boundary,
each ray of light will refract away from the normal to the surface. Since the light ray is passing
from a medium in which it travels slow (more optically dense) to a medium in which it travels
fast (less optically dense), it will bend away from the normal line; this is the SFA principle of
refraction.

The above diagram shows the behavior of two incident rays approaching parallel to the
principal axis. Note that the two rays converge at a point; this point is known as the focal
point of the lens. The first generalization which can be made for the refraction of light by a
double convex lens is as follows:

Refraction Rule for a Converging Lens


Any incident ray traveling parallel to the principal axis of a converging lens will refract through the lens and travel
through the focal point on the opposite side of the lens.

Now suppose that the rays of light are traveling through the focal point on the way to the lens.
These rays of light will refract when they enter the lens and refract when they leave the lens.
As the light rays enter into the more dense lens material, they refract towards the normal; and
as they exit into the less dense air, they refract away from the normal. These specific rays will
exit the lens traveling parallel to the principal axis.
The above diagram shows the behavior of two incident rays traveling through the focal point
on the way to the lens. Note that the two rays refract parallel to the principal axis. A second
generalization for the refraction of light by a double convex lens can be added to the first
generalization.

Refraction Rules for a Converging Lens


• Any incident ray traveling parallel to the principal axis of a converging lens will refract through the
lens and travel through the focal point on the opposite side of the lens.
• Any incident ray traveling through the focal point on the way to the lens will refract through the lens
and travel parallel to the principal axis.

The Thin Lens Approximation

These two "rules" will greatly simplify the task of determining the image location for objects
placed in front of converging lenses. This topic will be discussed in the next part of Lesson 5.
For now, internalize the meaning of the rules and be prepared to use them. As the rules are
applied in the construction of ray diagrams, do not forget the fact that Snells' Law of refraction
of light holds for each of these rays. It just so happens that geometrically, when Snell's Law is
applied for rays which strike the lens in the manner described above, they will refract in close
approximation with these two rules. The tendency of incident light rays to follow these rules is
increased for lenses which are thin. For such thin lenses, the path of the light through the lens
itself contributes very little to the overall change in the direction of the light rays. We will use
this so-called thin-lens approximation in this unit. Furthermore, to simplify the construction of
ray diagrams, we will avoid refracting each light ray twice - upon entering and emerging from
the lens. Instead, we will continue the incident ray to the vertical axis of the lens and refract
the light at that point. For thin lenses, this simplification will produce the same result as if we
were refracting the light twice.
Rules of Refraction for Diverging Lenses

Now let's investigate the refraction of light by double concave lens. Suppose
that several rays of light approach the lens; and suppose that these rays of
light are traveling parallel to the principal axis. Upon reaching the front face of
the lens, each ray of light will refract towards the normal to the surface. At this boundary, the
light ray is passing from air into a more dense medium (usually plastic or glass). Since the
light ray is passing from a medium in which it travels relatively fast (less optically dense) into
a medium in which it travels relatively slow (more optically dense), it will bend towards the
normal line. This is the FST principle of refraction. This is shown for two incident rays on the
diagram below. Once the light ray refracts across the boundary and enters the lens, it travels
in a straight line until it reaches the back face of the lens. At this boundary, each ray of light
will refract away from the normal to the surface. Since the light ray is passing from a medium
in which it travels relatively slow (more optically dense) to a medium in which it travels fast
(less optically dense), it will bend away from the normal line. This is the SFA principle of
refraction. These principles of refraction are identical to what was observed for the double
convex lens above.

The above diagram shows the behavior of two incident rays approaching parallel to the
principal axis of the double concave lens. Just like the the double convex lens above, light
bends towards the normal when entering and away from the normal when exiting the lens.
Yet, because of the different shape of the double concave lens, these incident rays are not
converged to a point upon refraction through the lens. Rather, these incident rays diverge
upon refracting through the lens. For this reason, a double concave lens can never produce a
real image. Double concave lenses produce images which are virtual. This will be discussed in
more detail in the next part of Lesson 5. If the refracted rays are extended backwards behind
the lens, an important observation is made. The extension of the refracted rays will intersect
at a point. This point is known as the focal point. Notice that a diverging lens such as this
double concave lens does not really focus the incident light rays which are parallel to the
principal axis; rather, it diverges these light rays. For this reason, a diverging lens is said to
have a negative focal length.

The first generalization can now be made for the refraction of light by a double concave lens:

Refraction Rule for a Diverging Lens


Any incident ray traveling parallel to the principal axis of a diverging lens will refract through the lens and travel in line
with the focal point (i.e., in a direction such that its extension will pass through the focal point).

Now suppose that the rays of light are traveling towards the focal point on the way to the
lens. Because of the negative focal length for double concave lenses, the light rays will head
towards the focal point on the opposite side of the lens. These rays will actually reach the lens
before they reach the focal point. These rays of light will refract when they enter the lens and
refract when they leave the lens. As the light rays enter into the more dense lens material,
they refract towards the normal; and as they exit into the less dense air, they refract away
from the normal. These specific rays will exit the lens traveling parallel to the principal axis.

The above diagram shows the behavior of two incident rays traveling towards the focal point
on the way to the lens. Note that the two rays refract parallel to the principal axis. A second
generalization for the refraction of light by a double concave lens can be added to the first
generalization.

Refraction Rules for a Diverging Lens


• Any incident ray traveling parallel to the principal axis of a diverging lens will refract through the
lens and travel in line with the focal point (i.e., in a direction such that its extension will pass through
the focal point).
• Any incident ray traveling towards the focal point on the way to the lens will refract through the lens
and travel parallel to the principal axis.

A Third Rule of Refraction for Lenses

The above discussion focuses on the manner in which converging and diverging lenses refract
incident rays which are traveling parallel to the principal axis or are traveling through (or
towards) the focal point. But these are not the only two possible incident rays. There are a
multitude of incident rays which strike the lens and refract in a variety of ways. Yet, there are
three specific rays which behave in a very predictable manner. The third ray which we will
investigate is the ray which passes through the precise center of the lens - through the point
where the principal axis and the vertical axis intersect. This ray will refract as it enters and
refract as it exits the lens, but the net affect of this dual refraction is that the path of the light
ray is not changed. For a thin lens, the refracted ray is traveling in the same direction as the
incident ray and is approximately in line with it. The behavior of this third incident ray is
depicted in the diagram below.

Now we have three incident rays whose refractive behavior is easily predicted. These three
rays lead to our three rules of refraction for converging and diverging lenses. These three rules
are summarized below.

Refraction Rules for a Converging Lens


• Any incident ray traveling parallel to the principal axis of a converging lens will refract through the
lens and travel through the focal point on the opposite side of the lens.
• Any incident ray traveling through the focal point on the way to the lens will refract through the lens
and travel parallel to the principal axis.
• An incident ray which passes through the center of the lens will in affect continue in the same
direction that it had when it entered the lens.

Refraction Rules for a Diverging Lens


• Any incident ray traveling parallel to the principal axis of a diverging lens will refract through the
lens and travel in line with the focal point (i.e., in a direction such that its extension will pass through
the focal point).
• Any incident ray traveling towards the focal point on the way to the lens will refract through the lens
and travel parallel to the principal axis.
• An incident ray which passes through the center of the lens will in affect continue in the same
direction that it had when it entered the lens.

These three rules of refraction for converging and diverging lenses will be applied through the
remainder of this lesson. The rules merely describe the behavior of three specific incident rays.
While there are a multitude of light rays being captured and refracted by a lens, only two rays
are needed in order to determine the image location. So as we proceed with this lesson, pick
your favorite two rules (usually, the ones which are easiest to remember) and apply them to
the construction of ray diagrams and the determination of the image location and
characteristics.

Next Section: Image Formation Revisited


Jump To Lesson 6: The Eye
Image Formation by Lenses
Image Formation Revisited
One major principle discussed in both Unit 13 and Unit 14 of The Physics Classroom Tutorial is
the line of sight principle:

In order to view an object, you must sight along a line at that object; and when you do light
will come from that object to your eye along the line of sight.
This very principle is combined with rules of reflection and refraction in order to explain how
an image is formed and what the characteristics of such images will be.

Plane Mirror Image Formation

In the plane mirror Lesson of Unit 13, it was mentioned that an image is formed by a plane
mirror as light emanates from an object in a variety of directions. Some of this light reaches
the mirror and reflects off the mirror according to the law of reflection. Each one of these rays
of light can be extended backwards behind the mirror where they will all intersect at a point
(the image point). Any person who is positioned along the line of one of these reflected rays
can sight along the line and view the image - a representation of the object. Thus,
an image location is a location in space where all the reflected light appears to come from.
Since light from the object appears to diverge from this location, a person who sights along a
line at this location will perceive a replica or likeness of the actual object. In the case of plane
mirrors, the image is said to be avirtual image. Virtual images are images which are formed
in locations where light does not actually reach. Light does not actually pass through the
location on the other side of the plane mirror; it only appears to an observer as though the
light were coming from this position.

Curved Mirror Image Formation

We have also seen how images are created by the reflection of light off curved mirrors.
Suppose that a light bulb is placed in front of a concave mirror; the light bulb will emit light in
a variety of directions, some of which will strike the mirror. Each individual ray of light will
reflect according to the law of reflection. Upon reflecting, the light will converge at a point. At
the point where the light from the object converges, a replica or likeness of the actual object is
created; this replica is known as the image. Once the reflected light rays reached the image
location, they begin to diverge. The point where all the reflected light rays converge is known
as the image point. Not only is it the point where light rays converge, it is also the point where
reflected light rays appear to an observer to be coming from. Regardless of the observer's
location, the observer will see a ray of light passing through the real image location. To view
the image, the observer must line her sight up with the image location in order to see the
image via the reflected light ray. The diagram below depicts several rays from the object
reflecting from the mirror and converging at the image location. The reflected light rays then
begin to diverge, with each one being capable of assisting an individual in viewing the image
of the object.
For plane mirrors, virtual images are formed. Light does not actually pass through the virtual
image location; it only appears to an observer as though the light was emanating from the
virtual image location. The image formed by this concave mirror is a real image. When a real
image is formed, it still appears to an observer as though light is diverging from the real
image location. Only in the case of a real image, light is actually passing through the image
location.

Converging Lens Image Formation

Converging lenses can produce both real and virtual images while diverging images can only
produce virtual images. The process by which images are formed for lenses is the same as the
process by which images are formed for plane and curved mirrors. Images are formed at
locations where any observer is sighting as they view the image of the object through the
lens. So if the path of several light rays through a lens is traced, each of these light rays will
intersect at a point upon refraction through the lens. Each observer must sight in the direction
of this point in order to view the image of the object. While different observers will sight along
different lines of sight, each line of sight intersects at the image location. The diagram below
shows several incident rays emanating from an object - a light bulb. Three of these incident
rays correspond to ourthree strategic and predictable light rays. Each incident ray will refract
through the lens and be detected by a different observer (represented by the eyes). The
location where the refracted rays are intersecting is the image location.

In this case, the image is a real image since the light rays are actually passing through the
image location. To each observer, it appears as though light is coming from this location.
Diverging Lens Image Formation

Diverging lens create virtual images since the refracted rays do not actually converge to a
point. In the case of a diverging lens, the image location is located on the object's side of the
lens where the refracted rays would intersect if extended backwards. Every observer would be
sighting along a line in the direction of this image location in order to see the image of the
object. As the observer sights along this line of sight, a refracted ray would come to the
observer's eye. This refracted ray originates at the object, and refracts through the lens. The
diagram below shows several incident rays emanating from an object - a light bulb. Three of
these incident rays correspond to our three strategic and predictable light rays. Each incident
ray will refract through the lens and be detected by a different observer (represented by the
eyes). The location where the refracted rays are intersecting is the image location. Since
refracted light rays do not actually exist at the image location, the image is said to be a virtual
image. It would only appear to an observer as though light were coming from this location to
the observer's eye.

Images of Objects Which Do Not Occupy a Single Point

The above discussion relates to the formation of an image by a "point object" - in this case, a
small light bulb. The same principles apply to objects which occupy more than one point in
space. For example, a person occupies a multitude of points in space. As you sight at a person
through a lens, light emanates from each individual point on that person in all directions.
Some of this light reaches the lens and refracts. All the light which originates from one single
point on the object will refract and intersect at one single point on the image. This is true for
all points on the object; light from each point intersects to create an image of this point. The
result is that a replica or likeness of the object is created as we sight at the object through the
lens. This replica or likeness is the image of that object. This is depicted in the diagram below.

Now that we have discussed how an image is formed, we will turn our attention to the use of
ray diagrams to predict the location and characteristics of images formed
byconverging and diverging lenses.

Image Formation by Lenses


Converging Lenses - Ray Diagrams
One theme of the Reflection and Refraction units of The Physics Classroom Tutorial has been
that we see an object because light from the object travels to our eyes as we sight along a line
at the object. Similarly, we see an image of an object because light from the object reflects off
a mirror or refracts through a transparent material and travel to our eyes as we sight at the
image location of the object. From these two basic premises, we have defined the image
location as the location in space where light appears to diverge from. Because light emanating
from the object converges or appears to diverge from this location, a replica or likeness of the
object is created at this location. For both reflection and refraction scenarios, ray diagrams
have been a valuable tool for determining the path of light from the object to our eyes.

In this section of Lesson 5, we will investigate the method for drawing ray diagrams for objects
placed at various locations in front of a double convex lens. To draw these ray diagrams, we
will have to recall the three rules of refraction for a double convex lens:

• Any incident ray traveling parallel to the principal axis of a converging lens
will refract through the lens and travel through the focal point on the opposite
side of the lens.
• Any incident ray traveling through the focal point on the
way to the lens will refract through the lens and travel
parallel to the principal axis.
• An incident ray which passes through the center of the lens
will in effect continue in the same direction that it had when it entered the
lens.

Earlier in this lesson, the following diagram illustrating the path of light from an object through
a lens to an eye placed at various locations was shown.

In this diagram, five incident rays are drawn along with their corresponding refracted rays.
Each ray intersects at the image location and then travels to the eye of an observer. Every
observer would observe the same image location and every light ray would follow the Snell's
Law of refraction. Yet only two of these rays would be needed to determine the image location
since it only requires two rays to find the intersection point. Of the five incident rays drawn,
three of them correspond to the incident rays described by our three rules of refraction for
converging lenses. We will use these three rays through the remainder of this lesson, merely
because they are the easiest rays to draw. Certainly two rays would be all that is necessary;
yet the third ray will provide a check of the accuracy of our process.

Step-by-Step Method for Drawing Ray Diagrams

The method of drawing ray diagrams for double convex lens is described below. The
description is applied to the task of drawing a ray diagram for an object located beyondthe 2F
point of a double convex lens.

1. Pick a point on the top of the object and draw three incident rays traveling towards the lens.

Using a straight edge, accurately draw one ray so that it passes exactly through the focal point
on the way to the lens. Draw the second ray such that it travels exactly parallel to the principal
axis. Draw the third incident ray such that it travels directly to the exact
center of the lens. Place arrowheads upon the rays to indicate their
direction of travel.

2. Once these incident rays strike the lens, refract them according to the three rules of
refraction for converging lenses.

The ray that passes through the focal point on the way to the lens will refract and travel
parallel to the principal axis. Use a straight edge to accurately draw its path. The ray which
traveled parallel to the principal axis on the way to the lens will refract and travel through the
focal point. And the ray which traveled to the exact center of the lens will continue in the same
direction. Place arrowheads upon the rays to indicate their direction of travel. Extend the rays
past their point of intersection.

3. Mark the image of the top of the object.

The image point of the top of the object is the point where the three refracted rays intersect.
All three rays should intersect at exactly the same point. This point is merely the point where
all light from the top of the object would intersect upon refracting through the lens. Of course,
the rest of the object has an image as well and it can be found by applying the same three
steps to another chosen point. (See notebelow.)

4. Repeat the process for the bottom of the object.

One goal of a ray diagram is to determine the location, size, orientation, and type of image
which is formed by the double convex lens. Typically, this requires determining where the
image of the upper and lower extreme of the object is located and then tracing the entire
image. After completing the first three steps, only the image location of the top extreme of the
object has been found. Thus, the process must be repeated for the point on the bottom of the
object. If the bottom of the object lies upon the principal axis (as it does in this example),
then the image of this point will also lie upon the principal axis and be the same distance from
the mirror as the image of the top of the object. At this point the entire image can be filled in.

Some students have difficulty understanding how the entire image of an object can be
deduced once a single point on the image has been determined. If the object is merely a
vertical object (such as the arrow object used in the example below), then the process is easy.
The image is merely a vertical line. In theory, it would be necessary to pick each point on the
object and draw a separate ray diagram to determine the location of the image of that point.
That would require a lot of ray diagrams as illustrated in the diagram below.

Fortunately, a shortcut exists. If the object is a vertical line, then the image is also a vertical
line. For our purposes, we will only deal with the simpler situations in which the object is a
vertical line which has its bottom located upon the principal axis. For such simplified
situations, the image is a vertical line with the lower extremity located upon the principal axis.

The ray diagram above illustrates that when the object is located at a position beyondthe 2F
point, the image will be located at a position between the 2F point and the focal point on the
opposite side of the lens. Furthermore, the image will be inverted, reduced in size (smaller
than the object), and real. This is the type of information which we wish to obtain from a ray
diagram. These characteristics of the image will be discussed in more detail in the next section
of Lesson 5.

Once the method of drawing ray diagrams is practiced a couple of times, it becomes as natural
as breathing. Each diagram yields specific information about the image. The two diagrams
below show how to determine image location, size, orientation and type for situations in which
the object is located at the 2F point and when the object is located between the 2F point and
the focal point.
It should be noted that the process of constructing a ray diagram is the same regardless of
where the object is located. While the result of the ray diagram (image location, size,
orientation, and type) is different, the same three rays are always drawn. The three rules of
refraction are applied in order to determine the location where all refracted rays appear to
diverge from (which for real images, is also the location where the refracted rays intersect).

Ray Diagram for Object Located in Front of the Focal Point

In the three cases described above - the case of the object being located beyond 2F, the case
of the object being located at 2F,and the case of the object being located between 2F and F -
light rays are converging to a point after refracting through the lens. In such cases, a real
image is formed. As discussed previously, a real image is formed whenever refracted light
passes through the image location. While diverging lenses always produce virtual images,
converging lenses are capable of producing both real and virtual images. As shown above, real
images are produced when the object is located a distance greater than one focal length from
the lens. A virtual image is formed if the object is located less than one focal length from the
converging lens. To see why this is so, a ray diagram can be used.

A ray diagram for the case in which the object is located in front of the focal point is shown in
the diagram at the right. Observe that in this case the light rays diverge after refracting
through the lens. When refracted rays diverge, a virtual image is formed. The image location
can be found by tracing all light rays backwards until they intersect. For every observer, the
refracted rays would seem to be diverging from this point; thus, the point of intersection of
the extended refracted rays is the image point. Since light does not actually pass through this
point, the image is referred to as a virtual image. Observe that when the object in located in
front of the focal point of the converging lens, its image is an upright and enlarged image
which is located on the object's side of the lens. In fact, one generalization which can be made
about all virtual images produced by lenses (both converging and diverging) is that they are
always upright and always located on the object's side of the lens.
Ray Diagram for Object Located at the Focal Point

Thus far we have seen via ray diagrams that a real image is produced when an object is
located more than one focal length from a converging lens; and a virtual image is formed
when an object is located less than one focal length from a converging lens (i.e., in front of F).
But what happens when the object is located at F? That is, what type of image is formed when
the object is located exactly one focal length from a converging lens? Of course a ray diagram
is always one tool to help find the answer to such a question. However, when a ray diagram is
used for this case, an immediate difficulty is encountered. The diagram below shows two
incident rays and their corresponding refracted rays.

For the case of the object located at the focal point (F), the light rays neither converge nor
diverge after refracting through the lens. As shown in the diagram above, the refracted rays
are traveling parallel to each other. Subsequently, the light rays will not converge to form a
real image; nor can they be extended backwards on the opposite side of the lens to intersect
to form a virtual image. So how should the results of the ray diagram be interpreted? The
answer: there is no image!! Surprisingly, when the object is located at the focal point, there is
no location in space at which an observer can sight from which all the refracted rays appear to
be coming. An image cannot be found when the object is located at the focal point of a
converging lens.

Next Section: Converging Lenses - Object-Image Relations


Jump To Lesson 6: The Eye
Image Formation by Lenses
Converging Lenses - Object-Image Relations
Previously in Lesson 5, ray diagrams were constructed in order to determine the general
location, size, orientation, and type of image formed by double convex lenses. Perhaps you
noticed that there is a definite relationship between the image characteristics and the location
where an object placed in front of a double convex lens. The purpose of this portion of the
lesson is to summarize these object-image relationships. The best means of
summarizing this relationship is to divide the possible object locations into five general areas
or points:

• Case 1: the object is located beyond the 2F point


• Case 2: the object is located at the 2F point
• Case 3: the object is located between the 2F point and the focal point (F)
• Case 4: the object is located at the focal point (F)
• Case 5: the object is located in front of the focal point (F)

Case 1: The object is located beyond 2F


When the object is located at a location beyond the 2F point, the image will always be located
somewhere in between the 2F point and the focal point (F) on the other side of the lens.
Regardless of exactly where the object is located, the image will be located in this specified
region. In this case, the image will be an inverted image. That is to say, if the object is right-
side up, then the image is upside down. In this case, the image is reduced in size; in other
words, the image dimensions are smaller than the object dimensions. If the object is a six-foot
tall person, then the image is less than six feet tall. Earlier in Unit 13, the
term magnification was introduced; the magnification is the ratio of the height of the object
to the height of the image. In this case, the magnification is a number with an absolute value
less than 1. Finally, the image is a real image. Light rays actually converge at the image
location. If a sheet of paper was placed at the image location, the actual replica or likeness of
the object would appear projected upon the sheet of paper.

Case 2: The object is located at 2F


When the object is located at the 2F point, the image will also be located at the 2F point on
the other side of the lens. In this case, the image will be inverted (i.e., a right-side-up object
results in an upside-down image). The image dimensions are equal to the object dimensions.
A six-foot tall person would have an image which is six feet tall; the absolute value of the
magnification is exactly 1. Finally, the image is a real image. Light rays actually converge at
the image location. As such, the image of the object could be projected upon a sheet of paper.

Case 3: The object is located between 2F and F


When the object is located in front of the 2F point, the image will be located beyond the 2F
point on the other side of the lens. Regardless of exactly where the object is located between
C and F, the image will be located in the specified region. In this case, the image will be
inverted (i.e., a right-side-up object results in an upside-down image). The image
dimensions are larger than the object dimensions. A six-foot tall person would have an image
which is larger than six feet tall. The absolute value of the magnification is greater than 1.
Finally, the image is a real image. Light rays actually converge at the image location. As such,
the image of the object could be projected upon a sheet of paper.

Case 4: The object is located at F


When the object is located at the focal point, no image is formed. As discussed earlier in
Lesson 5, the refracted rays neither converge or diverge. After refracting, the light rays are
traveling parallel to each other and cannot produce an image.

Case 5: The object is located in front of F


When the object is located at a location in front of the focal point, the image will always be
located somewhere on the same side of the lens as the object. Regardless of exactly where in
front of F the object is located, the image will always be located on the object's side of the
lens and somewhere further from the lens. The image is locatedbehind the object. In this case,
the image will be an upright image. That is to say, if the object is right-side up, then the
image will also be right-side up. In this case, the image is enlarged; in other words, the
image dimensions are greater than the object dimensions. A six-foot tall person would have an
image which is larger than six feet tall. The magnification is greater than 1.Finally, the image
is a virtual image. Light rays diverge upon refraction; for this reason, the image location can
only be found by extending the refracted rays backwards on the object's side the lens. The
point of their intersection is the virtual image location. It would appear to any observer as
though light from the object were diverging from this location. Any attempt to project such an
image upon a sheet of paper would fail since light does not actually pass through the image
location.

It might be noted from the above descriptions that there is a relationship between the object
distance and object size and the image distance and image size. Starting from a large value,
as the object distance decreases (i.e., the object is moved closer to the lens), the image
distance increases; meanwhile, the image height increases. At the 2F point, the object
distance equals the image distance and the object height equals the image height. As the
object distance approaches one focal length, the image distance and image height approaches
infinity. Finally, when the object distance is equal to exactly one focal length, there is no
image. Then altering the object distance to values less than one focal length produces images
which are upright, virtual and located on the same side of the lens as the object. Finally, if the
object distance approaches 0, the image distance approaches 0 and the image height
ultimately becomes equal to the object height. These patterns are depicted in the diagram
below. Eight different object locations are drawn in red and labeled with a number; the
corresponding image locations are drawn in blue and labeled with the identical number.

Check Your Understanding

1. Identify the means by which you can use a converging lens to form a real image.

2. Identify the means by which you can use a converging lens to form a virtual image.

3. A converging lens is sometimes used as a magnifying glass. Explain how this works;
specifically, identify the general region where the object must be placed in order to produce
the magnified effect.

Image Formation by Lenses


Diverging Lenses - Ray Diagrams
Earlier in Lesson 5, we learned how light is refracted by double concave lens in a
manner that a virtual image is formed. We also learned about three
simple rules of refraction fordouble concave lenses:

• Any incident ray traveling parallel to the principal axis of a diverging lens will
refract through the lens and travel in line with the focal point (i.e., in a
direction such that its extension will pass through the focal point).
• Any incident ray traveling towards the focal point on the way to the lens will
refract through the lens and travel parallel to the principal axis.
• An incident ray which passes through the center of the lens will in affect
continue in the same direction that it had when it entered the lens.

These three rules will be used to construct ray diagrams. A ray diagram is a tool used to
determine the location, size, orientation, and type of image formed by a lens. Ray diagrams
for double convex lenses were drawn in a previous part of Lesson 5. In this lesson, we will see
a similar method for constructing ray diagrams for double concave lenses.

Step-by-Step Method for Drawing Ray Diagrams

The method of drawing ray diagrams for a double concave lens is described below.

1. Pick a point on the top of the object and draw three incident rays traveling towards the lens.

Using a straight edge, accurately draw one ray so that it travels towards the focal point on the
opposite side of the lens; this ray will strike the lens before reaching the focal point; stop the
ray at the point of incidence with the lens. Draw the second ray such that it travels exactly
parallel to the principal axis. Draw the third ray to the exact center of the lens. Place
arrowheads upon the rays to indicate their direction of travel.

2. Once these incident rays strike the lens, refract them according to the three rules of
refraction for double concave lenses.

The ray that travels towards the focal point will refract through the lens and travel parallel to
the principal axis. Use a straight edge to accurately draw its path. The ray which traveled
parallel to the principal axis on the way to the lens will refract and travel in a direction such
that its extension passes through the focal point on the object's side of the lens. Align a
straight edge with the point of incidence and the focal point, and draw the second refracted
ray. The ray which traveled to the exact center of the lens will continue to travel in the same
direction. Place arrowheads upon the rays to indicate their direction of travel. The three rays
should be diverging upon refraction.
3. Locate and mark the image of the top of the object.

The image point of the top of the object is the point where the three refracted rays
intersect. Since the three refracted rays are diverging, they must be extended behind the lens
in order to intersect. Using a straight edge, extend each of the rays using dashed lines. Draw
the extensions until they intersect. All three extensions should intersect at the same location.
The point of intersection is the image point of the top of the object. The three refracted rays
would appear to diverge from this point. This is merely the point where all light from the top of
the object would appear to diverge from after refracting through the double concave lens. Of
course, the rest of the object has an image as well and it can be found by applying the same
three steps to another chosen point. See note below.

4. Repeat the process for the bottom of the object.

The goal of a ray diagram is to determine the location, size, orientation, and type of
image which is formed by the double concave lens. Typically,
this requires determining where the image of the upper and
lower extreme of the object is located and then tracing the
entire image. After completing the first three steps, only the
image location of the top extreme of the object has been
found. Thus, the process must be repeated for the point on
the bottom of the object. If the bottom of the object lies
upon the principal axis (as it does in this example), then the
image of this point will also lie upon the principal axis and be
the same distance from the lens as the image of the top of
the object. At this point the complete image can be filled in.

Some students have difficulty understanding how the entire image of an object can be
deduced once a single point on the image has been determined. If the object is merely a
vertical object (such as the arrow object used in the example below), then the process is easy.
The image is merely a vertical line. This is illustrated in the diagram below. In theory, it would
be necessary to pick each point on the object and draw a separate ray diagram to determine
the location of the image of that point. That would require a lot of ray diagrams as illustrated
in the diagram below.
Fortunately, a shortcut exists. If the object is a vertical line, then the image is also a vertical
line. For our purposes, we will only deal with the simpler situations in which the object is a
vertical line which has its bottom located upon the principal axis. For such simplified
situations, the image is a vertical line with the lower extremity located upon the principal axis.

The ray diagram above illustrates that the image of an object in front of a double concave
lens will be located at a position behind the double concave lens. Furthermore, the image will
be upright, reduced in size (smaller than the object), and virtual. This is the type of
information which we wish to obtain from a ray diagram. The characteristics of this image will
be discussed in more detail in the next section of Lesson 5.

Once the method of drawing ray diagrams is practiced a couple of times, it becomes as natural
as breathing. Each diagram yields specific information about the image. It is suggested that
you take a few moments to practice a few ray diagrams on your own and to describe the
characteristics of the resulting image. The diagrams below provide the setup; you must merely
draw the rays and identify the image. If necessary, refer to the method described above.

Next Section: Diverging Lenses - Object-Image Relations


Jump To Lesson 6: The Eye
Image Formation by Lenses
Diverging Lenses - Object-Image Relations
Previously in Lesson 5, ray diagrams were constructed in order to determine the location, size,
orientation, and type of image formed by double concave lenses (i.e., diverging lenses). The
ray diagram constructed earlier for a diverging lens revealed that the image of the object was
virtual, upright, reduced in size and located on the same side of the lens as the object. But will
these always be the characteristics of an image produced by a double concave lens? Can
convex lenses ever produce real images? Inverted images? Magnified Images? To answer
these questions, we will look at three different ray diagrams for objects positioned at different
locations along the principal axis. The diagrams are shown below. (Note that only two sets of
incident and refracted rays were used in the diagram in order to avoid overcrowding the
diagram with rays.)

The diagrams above shows that in each case, the image is

• located on the object' side of the lens


• a virtual image
• an upright image
• reduced in size (i.e., smaller than the object)

Unlike converging lenses, diverging lenses always produce images which share these
characteristics. The location of the object does not affect the characteristics of the image. As
such, the characteristics of the images formed by diverging lenses are easily predictable.

Another characteristic of the images of objects formed by diverging lenses pertains to how a
variation in object distance effects the image distance and size. The diagram below shows five
different object locations (drawn and labeled in red) and their corresponding image locations
(drawn and labeled in blue).
The diagram shows that as the object distance is decreased, the image distance is decreased
and the image size is increased. So as an object approaches the lens, its virtual image on the
same side of the lens approaches the lens as well; and at the same time, the image becomes
larger.

Check Your Understanding

The following questions pertain to the image characteristics of all types of optical devices
discussed in the last two units - plane mirrors, concave mirrors, convex mirrors,converging
lenses, and diverging lenses. Use your understanding of the object-image relationships for
these three types of mirrors and two types of lenses to answer these questions.

1. How can a plane mirror, concave mirror, convex mirror, converging lens and/or diverging
lens be used to produce an image which has the same size as the object?

2. How can a plane mirror, concave mirror, convex mirror, converging lens and/or diverging
lens be used to produce a magnified image?
3. How can a plane mirror, concave mirror, convex mirror,
converging lens and/or diverging lens be used to produce an
upright image?

4. How can a a plane mirror, concave mirror, convex mirror, converging lens and/or diverging
lens be used to produce a real image?

5. The image of an object is found to be upright and reduced in size. What type of mirror
and/or lens is used to produce such an image?

The Eye
The Anatomy of the Eye
The human eye is a complex anatomical device that remarkably demonstrates the
architectural wonders of the human body. Like a camera, the eye is able to refract light and
produce a focused image that can stimulate neural responses and enable the ability to see. In
Lesson 6, we will focus on the physics of sight. We will use our understanding of refraction and
image formation to understand the means by which the human eye produces images of distant
and nearby objects. Additionally, we will investigate some of the common vision problems
which plague humans and the customary solutions to those problems. As we proceed through
Lesson 6, we will apply our understanding of refraction and lenses to the physics of sight.

The eye is essentially an opaque eyeball filled with a water-like fluid. In the front of the eyeball
is a transparent opening known as the cornea. The cornea is a thin membrane which has an
index of refraction of approximately 1.38. The cornea has the dual purpose of protecting the
eye and refracting light as it enters the eye. After light passes through the cornea, a portion
of it passes through an opening known as the pupil. Rather than being an actual part of the
eye's anatomy, the pupil is merely an opening. The pupil is the black portion in the middle of
the eyeball. It's black appearance is attributed to the fact that the light which the pupil allows
to enter the eye is absorbed on the retina (and elsewhere) and does not exit the
eye. Thus, as you sight at another person's pupil opening, no light is exiting their
pupil and coming to your eye; subsequently, the pupil appears black.

Like the aperture of a camera, the size of the pupil opening can be adjusted by the dilation of
the iris. The iris is the colored part of the eye - being blue for some people and brown for
others (and so forth); it is a diaphragm which is capable of stretching and reducing the size of
the opening. In bright-light situations, the iris adjusts its size to reduce the pupil opening and
limit the amount of light which enters the eye. And in dim-light situations, the iris adjusts so
as to maximize the size of the pupil opening and increase the amount of light which enters the
eye.

Light which passes through the pupil opening, will enter the crystalline lens. The crystalline
lens is made of layers of a fibrous material which has an index of refraction of roughly 1.40.
Unlike the lens on a camera, the lens of the eye is able to change its shape and thus serves to
fine-tune the vision process. The lens is attached to theciliary muscles. These muscles relax
and contract in order to change the shape of the lens. By carefully adjusting the lenses shape,
the ciliary muscles assist the eye in the critical task of producing an image on the back of the
eyeball.

The inner surface of the eye is known as the retina. The retina contains the rods and cones
which serve the task of detecting the intensity and the frequency of the incoming light. An
adult eye is typically equipped with up to 120 million rods which detect the intensity of light
and about 6 million cones which detect the frequency of light. These rods and cones send
nerve impulses to the brain. The nerve impulses travel through a network of nerve cells. There
are as many as one-million neural pathways from the rods and cones to the brain. This
network of nerve cells is bundled together to form the optic nerve on the very back of the
eyeball.

Each part of the eye plays a distinct part in enabling humans to see. The ultimate goal of such
an anatomy is to allow humans to focus images on the back of the retina. This task is
discussed in the next part of Lesson 6.

The Eye
Image Formation and Detection
Earlier in Lesson 6, we learned that the eye consists of a cornea (thin outer membrane), a lens
attached to ciliary muscles, and a retina (inner surface equipped with nerve cells). These four
parts of the eye are the most instrumental in the task of producing images which are
discernible by the brain. In order to facilitate the ability to see, each part must enable the eye
to refract light so that is produces a focused image on the retina.

It is a surprise to most people to find out that the lens of the eye is not where all the
refraction of incoming light rays takes place. Most of the refraction occurs at the cornea. The
cornea is the outer membrane of the eyeball which has an index of refraction of 1.38. The
index of refraction of the cornea is significantly greater than the index of refraction of the
surrounding air. This difference in optical density between the air the corneal material
combined with the fact that the cornea has the shape of a converging lens is what explains the
ability of the cornea to do most of the refracting of incoming light rays. The crystalline lens is
able to alter its shape due to the action of the ciliary muscles. This serves to induce small
alterations in the amount of corneal bulge as well as to fine-tune some of the additional
refraction which occurs as light passes through the lens material.

The bulging shape of the cornea causes it to refract light in a manner to similar to adouble
convex lens. The focal length of the cornea-lens system varies with the amount of contraction
(or relaxation) of the ciliary muscles and the resulting shape of the lens. In general, the focal
length is approximately 1.8 cm, give or take a millimeter. As learned in our discussion
of convex lenses in Lesson 5, the image location, size, orientation, and type is dependent upon
the location of the object relative to the focal point and the 2F point of a lens system. Since
the object is typically located at a point in space more than 2-focal lengths from the "lens,"
the image will be located somewhere between the focal point of the "lens" and the 2F point.
The image will be inverted, reduced in size, and real. Quite conveniently, the cornea-lens
system produces an image of an object on the retinal surface. The process by which this
occurs is known as accommodation and will be discussed in more detail in the next part of
Lesson 6. Fortunately, the image is a real image - formed by the actual convergence of light
rays at a point in space. Vision is dependent upon the stimulation of nerve impulses by an
incoming light rays. Only real images would be capable of producing such a stimulation.
Finally, the reduction in the size of the image allows the entire image to "fit" on the retina. The
fact that the image is inverted poses no problem. Our brain has become quite accustomed to
this and properly interprets the signal as originating from a right-side-up object.
The use of the lens equation and magnification equation can provide an idea of the
quantitative relationship between the object distance, image distance and focal length. For
now we will assume that the cornea-lens system has a focal length of 1.80 cm (0.0180 m). We
will attempt to determine the image size and image location of a 6-foot tall man (ho=1.83 m)
who is standing a distance of approximately 10 feet away (do= 3.05 meters). (The lens
equation is derived geometrically upon the assumption that the lens is a thin lens. The lens of
the eye is anything but thin and as such the lens equation does not provide a truly accurate
model of the eye lens. Despite this fact, we will use the equation as a simplified approximation
of the mathematics of the eye.)

Like all problems in physics, begin by the identification of the known information.

do = 3.05 m ho = 1.83 m f = 0.0180 m

Next identify the unknown quantities which you wish to solve for.

di = ??? hi = ???

To determine the image distance, the lens equation can be used. The following lines represent
the solution to the image distance; substitutions and algebraic steps are shown.

1/f = 1/do + 1/di


1/(0.0180 m) = 1/(3.05 m) + 1/di

55.6 m-1 = 0.328 m-1 + 1/di

55.2 m-1 = 1/di

di = 0.0181 m = 1.81 cm

To determine the image height, the magnification equation is needed. Since three of the four
quantities in the equation (disregarding the M) are known, the fourth quantity can be
calculated. The solution is shown below.

hi/ho = - di/do
hi /(1.83 m) = - ( 0.0181 m)/(3.05 m)

hi = - (1.83 m) • ( 0.0181m)/(3.05 m)

hi = -0.0109 m = -1.09 cm

From the calculations in this problem it can be concluded that if a 1.83-m tall person is
standing 3.05 m from your cornea-lens system having a focal length of 1.8 cm, then the
image will be inverted, 1.09-cm tall (the negative values for image height indicate that the
image is an inverted image) and located 1.81 cm from the "lens".

Now of course, if the person is standing further away (or closer to) the eye, the image size
and image distance will be adjusted accordingly. This is illustrated in the following table for the
same 6-foot tall (1.83 m) person.

Dependence of himage and dimage on dobject


(focal length is fixed at 1.8 cm)
Object Distance Image Distance Image Height
1.00 m 1.83 cm 3.35 cm
3.05 m 1.81 cm 1.09 cm
100 m 1.80 cm 0.329 cm

The results of these calculations (as illustrated in the table above) illustrate two important
principles concerning the ability of the eye to discern objects which are both close up and far
away. First, the distance between the observer and the object will greatly influence the image
size (height and width of the image formed on the retina) and quality. Objects which are
viewed at close proximity produce images which are larger than distant objects. Such an
image is typically spread over the entirety of the retina and even at times would extend
beyond the extremities of the retina. Thus, the full dimensions of a 6-foot tall person cannot
be seen if he/she stands 1-meter away (unless the eyeballs are rolled in their socket). On the
other hand, the entire image of the same person can easily be seen when standing 100-
meters away. In this instance, the image takes up a small amount of space on the back of the
retina. The drawback however is that the finer details of the image are lost due to the
reduction in the size of the image. Since the image is stimulating a smaller region of nerve
cells, some details are lost since they fail to provide sufficient stimulation to allow the brain to
discern them. For such an object distance, the small image which is created of a man might
make it impossible to discern that the man's fly is open or that his socks and belt don't match.

Second, the varying distance between the observer and the object poses some potential
problems for the human eye. Objects located varying distances from a lens system with a
fixed focal length produce images which are varying distances from the lens. Yet, the eye must
always produce an image on the retina - a location which is always the same distance away
from the cornea. The eye cannot afford to allow changes in the image distance. So how does
an eye always focus images with the same dimage regardless of the fact that the dobject is
different? How can an object 100 meters away be focused the same distance from the cornea-
lens system as an object which is 1 meter away? The answer: the cornea-lens system is able
to change its focal length. The ciliary muscles of the eye serve to contract and relax, thus
changing the shape of the lens. This serves to to allow the eye to change its focal length and
thus appropriately focus images of objects which are both close up and far
away. This process is known as accommodation and is the focus of the next
part of Lesson 6.

The Eye
The Wonder of Accommodation
While the entire surface of the retina contains nerve cells, there is a small portion with a
diameter of approximately 0.25 mm where the concentration of rods and cones is greatest.
This region, known as the fovea centralis, is the optimal location for the formation of the
image. The eye typically rotates in its socket in order to focus images of objects at this
location. The distance from the outer surface of the cornea (where the light undergoes most of
its refraction) to the central portion of the fovea on the retina is approximately 2.4 cm. Light
entering the cornea must produce an image with a distance of 2.4 cm from its outer edge.
Unlike a camera, which has the ability to change the distance between the film (the detector)
and the lens, the distance between the retina (the detector) and the cornea (the refractor) is
fixed. The image distance is unchangeable. Subsequently, the eye must be able to alter the
focal length in order to focus images of both nearby and far away objects upon the retinal
surface. As the object distance changes, the focal length must be changed in order to keep the
image distance constant.

The ability of the eye to adjust its focal length is known as accommodation. Since a nearby
object (small dobject) is typically focused at a further distance (large dimage), the eye
accommodates by assuming a lens shape that has a shorter focal length. This reduction in
focal length will cause more refraction of light and serve to bring the image back closer to the
cornea/lens system and upon the retinal surface. So for nearby objects, the ciliary
muscles contract and squeeze the lens into a more convex shape. This increase in the
curvature of the lens corresponds to a shorter focal length. On the other hand, a distant object
(large dobject) is typically focused at a closer distance (small dimage). The eye accommodates by
assuming a lens shape that has a longer focal length. So for distant objects the ciliary
muscles relax and the lens returns to a flatter shape. This decrease in the curvature of the
lens corresponds to a longer focal length. The data table below demonstrates how a changing
focal length would be required to maintain a constant image distance of 1.80 cm.

Dependence of f upon dobject


(dimage is fixed at 1.80 cm)
Object Distance Focal Length
0.25 m 1.68 cm
1m 1.77 cm
3m 1.79 cm
100 m 1.80 cm
Infinity 1.80 cm
(The values above have been calculated using the lens equation. The lens equation represents a simplified mathematical
model of the eye.)
The ability of the eye to accommodate is automatic. Furthermore, it occurs
instantaneously. Focus on a far away object and quickly turn your attention
to a nearby object; observe that there is no noticeable delay in the ability of the eye to bring
the nearby object into focus. Accommodation is a remarkable feat!

The power of a lens is measured by opticians in a unit known as a diopter. A diopter is the
reciprocal of the focal length.

diopters = 1/(focal length)

A lens system with a focal length of 1.8 cm (0.018 m) is a 56-diopter lens. A lens system with
a focal length of 1.68 cm is a 60-diopter lens. A healthy eye is able to bring both distant
objects and nearby objects into focus without the need for corrective lenses. That is, the
healthy eye is able to assume both a small and a large focal length; it would have the ability
to view objects with a large variation in distance. The maximum variation in the power of the
eye is called the Power of Accommodation. If an eye has the ability to assume a focal
length of 1.80 cm (56 diopters) to view objects many miles away as well as the ability to
assume a 1.68 cm focal length to view an object 0.25 meters away (60 diopters), then its
Power of Accommodation would be measured as 4 diopters (60 diopters - 56 diopters).

The healthy eye of a young adult has a Power of Accommodation of approximately 4 diopters.
As a person grows older, the Power of Accommodation typically decreases as a person
becomes less able to view nearby objects. This failure to view nearby objects leads to the
need for corrective lenses. In the next two sections of Lesson 6, we will discuss the two most
common defects of the eye - nearsightedness and farsightedness.

The Eye
Nearsightedness and its Correction
The human eye's ability to accommodate allows it to view focused images of both nearby and
distant objects. As mentioned earlier in Lesson 6, the lens of the eye assumes a large
curvature (short focal length) to bring nearby objects into focus and a flatter shape (long focal
length) to bring a distant object into focus. Unfortunately, the eye's inability to a provide a
wide variance in focal length leads to a variety of vision defects. Most often, the defect occurs
at one end of the spectrum - either the inability to assume a short focal length and focus on
nearby objects or the inability to assume a long focal length and thus focus on distant objects.
Nearsightedness or myopia is the inability of the eye to focus on distant objects. The
nearsighted eye has no difficulty viewing nearby objects. But the ability to view distant objects
requires that the light be refracted less. Nearsightedness will result if the light from distant
objects is refracted more than is necessary. The problem is most common as a youth, and is
usually the result of a bulging cornea or an elongated eyeball. If the cornea bulges more than
its customary curvature, then it tends to refract light more than usual. This tends to cause the
images of distant objects to form at locations in front of the retina. If the eyeball is elongated
in the horizontal direction, then the retina is placed at a further distance from the cornea-lens
system. Subsequently the images of distant objects form in front of the retina. On the retinal
surface, where the light-detecting nerve cells are located, the image is not focused. These
nerve cells thus detect a blurry image of distant objects.

The cure for the nearsighted eye is to equip it with a diverging lens. Since the nature of the
problem of nearsightedness is that the light is focused in front of the retina, a diverging lens
will serve to diverge light before it reaches the eye. This light will then be converged by the
cornea and lens to produce an image on the retina.

(Note: In the diagram above that the light approaching the eye from a distant object is
traveling as a bundle of rays which are roughly parallel to each other. Suppose for a moment
that the distant object is the lettering on the chalk board in the front of the room as you sight
at it from the back of the room. Geometrically, whatever light rays from a particular letter or
word which reach your eye will be traveling roughly parallel to each other. This is not the case
when viewing nearby objects as demonstrated on the previous page.)

You might also like